wk3 uworld

Pataasin ang iyong marka sa homework at exams ngayon gamit ang Quizwiz!

WK3---WED--->---mom brings her 19yo son to the office bc he's depressed..she wants him to get help but is concerned about him taking antidepressants dt the risk of suicide. -most appropriate response to the mom?

- The slightly increased risk of suicidal thoughts with antidepressant in this age group needs to be weighed against the risk of completed suicide in untreated depression. The slightly increased risk of antidepressant related suicidal thoughts and behavior and some children and adolescents must be weighed against efficacy of antidepressant and the risk of completed suicide and depression. Patient should be carefully monitored for suicidality at the beginning of antidepressant therapy.

WK3---SUN--->---23 yo f w/ pmh of T1dm...BMI and vitals goo...US shows twins..labs normal -nbsim?

-24hr urine collection for total protein pt is high risk for preeclampsia being diabetic put her at risk for nephorpathy and vascular dz..inc risk preeclampsia..and multiple gestations put her at risk too EO: For patients at high risk for preeclampsia such as type one diabetes multiple gestation, a 24 hour urine collection for a total protein at the initial prenatal visit establishes baseline renal function and dictates management such as delivery timing if hypertension develops later in pregnancy.

WK3---SUN--->---mom has tuberous sclerosis and dad is heathy -chances of them having a boy w/ the dz?

-50% chance bc its an autsomsomal dominant ...and mom is heterozygous for hte dz

WK3---WED--->--10yr old girl brought to office by mom b/c disruptive in school...forgets to do assignments...blurts out whatever comes to mind...doesnt follow directions in school...easily distracted.. -likely dx?

-ADHD The patient blurting out to communicate, disorganized schoolwork, difficulty following directions, and distractibility or characteristics of ADHD. Her inattentive symptoms include losing assignments struggle to follow directions forgetting to bring home books, not following through on homework, and being distractible. Hyper active, impulsive symptoms include and others and blurting out in our thoughts. ADHD may be mistaken for willfully defiant behavior and causes difficulties in social relationships. Disruptive mood dysregulation disorder involves poor frustration tolerance and persistent irritability that result in frequent temper outbursts verbal or physical out of proportion to the situation Conduct disorder patients experience behaviors that intentionally violate the rights of others a major society norms such as aggression property destruction theft and lying. CONDUCT for CHILDREN and ANTI-SOCIAL PERSONALITY DISORDER for ADULTS Oppositional defiant disorder is characterized by a pattern of angry/irritable mood, argumentative defiant behavior and vindictiveness that is deliberate in nature.

WK3---FRI--->--researchers comparing the clinical significance bt 3 different NSAID medications in pts w/ OA..group 1 got 400mg of drugA....group2 got 8mg of drugB...and group3 got 50mg of drugC. the primary endpoint was clinical improvement in pain,stiffness, and physical stiffness and physical fxn was measured on a WOMAC scale. Higher scores indicate worse pain..stiffness..and functional limitations. -which statistical is most appropriate to compare WOMAC scores among treatment groups?

-ANOVA -Anova is best bc its used for numerical date to compare the means of 3 or more groups..in this case would be the 3 types of meds -chi-sqare would not be the best bc its used to find associations and for categories/non numerical...such as gender..race..high v low -t-test coma=pares the mean of 2 group and is used for numerical data

WK3---THURS--->--28 yo F brought to the ED for bizarre behavior for the past week...pt abruptly quit her job as congressional aid bc she thinks boss is trying to sabotage her..she plans to run for president..boss is jealous of her...she hugs her boyfriend and says she cant live without him...rarely drinks and vitals are normal -likely dx?

-Bipolar -pt w/ 1 or more manic episode are diagnosed w/ bipolar1....depressive episodes are not required for diagnosis but are common...manic episodes may include psychotic sxs, often w/ grandiose themes.

WK3---WED--->--46 yo M present w/ sxs of depression..low energy and problem concentrating at work..gained 10 pound..tried escitalopram and sertraline but didnt help Which medication is most appropriate for this pt?

-Bupropion - Although SSRIs are common initial choice in the treatment of major depressive disorder many patients do not respond to them. This patient has failed two trials of antidepressants from the same class making it preferable to switch him to an antidepressant with a different mechanism of action because no antidepressant is more effective than others, drugs selection is based on factors such as safety side profile specific depressive symptoms for mobility and personal family history of antidepressant response. Alternate first line antidepressant medication's include BUPROPION, MIRTAZAPAINE, SSRI, BUPROPION is It is activating, does not cause weight gain, it has no sexual side effects. It would be a preferred choice for this patient who is depressive symptoms include hypersomnia and weight gain. Tricyclic antidepressants are generally reserved for treatment refractory patients such as not responded to multiple in different classes of first line antidepressants due to their less favorable side effect profile and safety issues such as cardiotoxicity and lethality in overdose

WK3---FRI--->--64 yo M present w/ anxiety an low mood for the past 2 months...not eating...recent DM dx but MBP is 19...20lb weight loss..pmh of smoking -nbsim?

-CT of abdomen bc pt has red flags pf pancreatic cancer..such as new dx of DM when pt is not overweight...thin..older...weight loss...smoking hx..depression that could be prodromal for pancreatic cancer -depression, weight loss and new onset of dm may occur as early manifestation of pancreatic cancer..CT of the abdomen is indicated as part of the initial diagnostic evaluation.

WK3---THURS--->--52yo M presents to the office requesting a refill for opioid prescriptint that doesnt run out for another 3 days...pt urine screen was positive for PCP but pt says he doesnt use any recreational drugs.. -most appropriate response?

-I need to obtain an additional urine study today and will call u to discuss the results -urine drug screening can be used in ptS on long term opoid therapy to detect recreational drug use...verify adherence to treatment and identify possible medication diversion. However, false positive results can occur ;unexpected results require confirmatory testing

WK3---THURS--->--27yo M is stable and has returned from hospital after hx of depression w/ suicide attempt...he was prescribed a medication that has created a macuopapular rash on his back -which med is responsible? -Lamotrigine

-Lamotrigine -mild rash may develop in up to 10% of those treated w/ lamotrigine...where threatening step Johnson syndrome or toxic epidermal necrolysis may occur in 0.1%. Any occurrence of rash during the treatment of lamotrigine requires immediate discontinuation

WK3---WED--->--27yo M presents w/ husband stating he has not been himself lately...he has been speaking rapidly that he has a plan to achieve world peace..he has experienced irritable mood...inc energy over the past 2 yrs.. -which combination medications is the most appropriate?

-Lithium +quetiapine The patient's current major depressive episode and history of manic episodes meet the criteria for bipolar one disorder. Bipolar disorder is a lifelong and highly recurrent illness that requires maintenance treatment to delay or prevent occurrence of new mood episodes, reduce the risk of suicide, and improve psychosocial fun functioning. Evidence based maintenance manual therapy options for bipolar providing quetiapine..lamotrigine however, those with severe illness as seen in this patient such as psychosis aggression frequent episodes of hospitalizations often require combination therapy to maintain stability. Lithium or VALPROATE combined with a second generation antipsychotic QUETIAPINE is first line combination near me. Bipolar disorder is a highly recurrent illness that requires maintenance pharmacotherapy. In patients with severe and frequent mood episodes, the combination of lithium or valproate and the second generation antipsychotic is used as first line combination pharmacotherapy.

WK3---THURS--->--63 yo M presents to the ED w/ AMS...temp 101...bp=160/90...agitated..confused...hyperreflexia...ankle clonus...after being prescribe dextromethorphan for a URI...pmh of MDD that he's taking meds for -initial tx?

-Lorazepam - This patient taking an SSRI has altered mental status, autonomic instability such as hyperthermia hypertension tachycardia dilated pupils and neuromuscular excitability such as hyperreflexia and ankle clonus, a triad concerning for serotonin syndrome. Although it's typically follows increase SSRI dosage, serotonin syndrome can also be precipitated by the use of another serotoninergic agent. This patient recently began taking dextromethorphan which primarily activates sigma opioid receptors to inhibit cough but it also has serotoninergic activity at 5HTA to receptors management includes discontinuation of all serotoninergic medication Management of severe hypertension with short acting anti-hypertensive such as esmolol. hydration and evaporated and convective cooling paragraph Benzodiazepines such as lorazepam to improve agitation, decrease muscle contraction which reduces hypothermia and decrease blood pressure and heart rate In patients who are on responsive to benzodiazepines and supportive care, CYPROHEPTADA in E is used to lower the serotonergic activity.

WK3---THURS--->--16yo F present w/ mom to the discuss her behavior...no longer interested in drama club or meeting w/ friends...sleep alot but still tired...fights more w/ siblings,...fidgety..distracted..fatigue -likely dx?

-MDD -Adolescents w/ depression may be characterized by irritability..loss of interest in activities previously enjoyed..fatigue..disturbance in sleep...comcemtration issues and appetite changes...excessive guilt/feeling worthless...suicidal ideation...the severity of the sxs and related impairment can help differentiate depression from adjustment disorder and normal adolescent behavior

WK3---THURS--->--28 yo woman present to the ED after having a generalized tonic-clonic seizure. Pt has been taking meds for her insomnia and anxiety for the past 6 moths and abruptly stopped talking it 2 days ago. -which med caused these sxs?

-alprazolam Abrupt cessation of alprazolam a short acting benzodiazepine is associated with significant withdrawal symptoms including a risk for generalized seizures and confusion. Abrupt discontinuation of paroxitene and venlafaxine has been associated with antidepressant discontinuation syndrome such as acute onset of dysphoria, fatigue, dizziness, gastrointestinal distress, flu like symptoms.

WK3---THURS--->--16yo boy presents to the ED after friends says he began having sever anxiety at a party..he was rocking back and forth..very withdrawn...feel like he cant breathe..like he's going to die...bp=140./80...pulse=110...RR=18...PE reveals anxious..withdrawn...3mm pupil..BL conjunctiva injection... -drug thats consistent w/pts presentation?

-cannabis intoxication -pt w/ cannabis intoxication present w/ BL conjunctival injection, dry mouth...tachycardia..inc appetite..psychomotor impairment such as anxiety..and paranoia may also occur at high doses -bath salts intoxication present w/ tacky cardia...htn...dilated pupil...agitation..and violent behavior -opoid tria—resp depression..cns depression..pin point pupils

WK3---FRI--->--55yo M has a rare cancer...he has Dr about risk factor that may have triggered it? -0best design of study?

-case controls -it retrospectively views data looing for the odds of a previous exposure on the development of a rare dz manifestation -it starts w/ ppl w/ the dz...and look back at other groups that are otherwise matched to asses for risk of exposure -case control is subject to recall bias -cohort initially identifies ppl based on exposure status to a risk factor and then follows both groups over time to assess development of dz. Its observational and prospective..there is no interventions..uses relative risk calculation to assess results. Not a good design to study rarew diseases w/ long latent period

WK3---SAT--->----40 yo F presents for first prenatal visit at 10wks GA..US reveals fetal pole at 10wks gestation w/ hr 160/min.. In addition to routine lab testing...which test is also recommended?

-cell-free-DNA test -women 35 and older are at inc risk of fetal aneuploidy and offered cell-free DNA testing of maternal plasma. This test is non invasive and highly sensitive and specific screening for fetal aneuploidy/chromosomal abnormalities..Its performed 10wk or higher...abnormal results are confirmed by chorionic villus sampling or amniocentesis

WK3---FRI--->--mom brings baby to dr appoint ..concerned about baby's accelerated weight gain and wondering if its linked to childhood obesity -best study to answer her question?

-cohort -cohort is best bc it follows pts over yrs to record the incidence of dz...its a model that is prospective and uses RELATIVE RISK assess results

WK3---SUN--->---Gall stone dz

-commonly develop in pregnancy bc elevated progesterone levels inhibit gb contraction and promote stone formation...inc the risk of cholecystitis and ascending cholangitis..pts typically presents w/ fever..abd pain..leukocytosis...gb wall thickening.. and elevate aminotransferases

WK3---WED--->--27 yo F presents w/ abd pain and high level of stress...since her breakup w/ her bf..she doesnt know how to go on w/out him bc he did everything for her such as manage finances...help her apply for a job..just moved in w/ a friend to seek help w/sorting things out -likely explanation for behavior?

-dependent personality disorder Dependent personality disorder is characterized by excessive dependent and submissive behaviors, indecisiveness, and fear of being left to manage alone.

WK3---SUN--->---32 yo F present at 16wks GA w/ complaints of inc thirst and urinary freq...urinatng 6 to 8 times/night...UA show specific gravity is 1.001(low is below 1.006..indicating dilute urine)...24hr urine collection test shows urine output of 5.5L(polyuria is more than 3L of urinary output in a 24hr period) after an intake of 2L. Likely dx?

-diabetes insipidus -polyuria...dilute urine/low specific gravity indicate diabetes insidious..despite normal serum sodium concentration and fluid intake...during pregnancy...preexisting mild DI sxs can worsen dt inc turnover of antidiuretic hormones

WK3---FRI--->--researchers collected county data on life expectancy from the CDC...and altitude data from the US geological survey -best describes the design of this study?

-ecological study -uses populations level rather than individual level date as a unit of analysis in such a manner is consistent w/ an ecological study.' -good for generating hypotheses but shuld not be used to draw conclusion regarding individuals w/in these populations education objection: in ecological studies..the unit of observation is the population, not the individual. Disease rates and exposures are measured in each of a series of populations and their associations determined. Results about associations at the population level may not translate to the individual

WK3---WED--->--68 yr old man is post op...w/ sxs of delirium...he states he's frightened by the small animals hopping around his bed -best initial intervention to prevent self harm?

-employ a professional sitter at bedside - Non-pharmacological management of delirium symptoms is preferred whenever possible. Personal interaction and constant observation reduce the need for physical restraints and can be provided by family members or professional sitters.

WK3---SAT--->---28 yo F w/ diffic conceiving..irregular cycles....labs show elevated testosterone..bf normal semen analysis... -likely cause?

-failure of follicle maturation -PT hasPCOS w/ eo of irregular cyc;les...elevated testosterone..infertility -There pt has infertility..irregular cycles...and hyperandrogegism..features that support PCOS...hyperandrogenism, resulting from ovarian steroid overproduction can be dx clinically (hirsituism...acne..alopecia) or biochemically w/ eye serum testoster. -POCOS is characterized by irregular menses and clinically or biochemically eo hyperandrogegism. PTS w/ POCOS may have infertility dt an ovulation from failed follicular maturation Primary ovar, insufficiency—presents w/ infertility and oligomenorrhea..pts usually have an autoimmune dz...normal testosterone levels...its also assoc w/ turners

WK3---SUN--->---35yo F presents for 1st prenatal visit..last cycle was 7wks ago..she had a positive pregnancy test a few days ago along w/ nausea and vomi...but it has subsided...she had one full term delivery follow ed by 2 miscarriages...6months ago she had sudden right arm weakness and slurred speech..which resolved spontaneously w/in 2hrs...US reveals and enlarged empty uterus -likely cause of miscarriage?

-hypercoaguable state -3 consistent miscarriages is consistent w/ recurrent pregnancy loss...also the episode of slurred speech w/ arm weakness is e/o of a transient ischemia attack..these clues point towards antiphospholipid syndrome..which is an autoimmune disorder cause by antiphopholipid antibodies such as lupus anticoagulant and antcardiolpin antibody that create a hypercoagualbe state leading to venous and arterial thrombosis such as TIA. APS requires a life long of anticoagulants..treatment is w/ heparin and warfarin..but I'm sure on heparin is used is female is pregnant bc warfarin is teratogen

WK3---WED--->--What kind of hormonal abnormalities is expected in Pts w/ MDD?

-inc serum cortisol concentration - Major depressive disorder is associated with hyperactivity of the hypothalamic-pituitary-adrenal axis, resulting in increased cortisol levels. Dec REM latency and dec slow-wave sleep is also expected.

WK3---SUN--->---26yo F w hx off hypothyroidism..talks to dr about adjusting medications bc planning for pregancy -best recommendation?

-inc the levothyroxine doss when the pt becomes pregnant -during pregnancy..estrogen induces an inc in serum thyroxine binding globulin levels..requiring an inc in the alt of thyroid hormone needed to saturate the binding sites...thyroxin production also inc dt stimulatory effect of hCG on TSH receptors. These changes lead to an inc in total thyroid hormone levels w/ minimal inc in free hormone levels and dec TSH...however..pt w/ pre-existing hypothyroidism are unable to inc thyroxine production appropriately ..at risk for worsening hypothyroid state and adverse fetal and maternal effects(gestational htn..preeclampsia..premature delivery...post partum hemorrhage) EO:levothyroxine requirements inc during pregnancy...pts w/ hypothyroidism should; inc dose at the time suspecting pregnancy...w/ subsequent dose adjustments based on TSH and total T4 pregnancy specific norms.

WK3---THURS--->--4yo boy present w/ parents bc he get frustrated bc no one can understand what he says...speaks in 2 or 3 word sentences..unable to name colors or letters...he says"school open Monday...not dog today" -likely dx?

-language disorder..think about toddler age when youre learning language - Language disorder becomes evident in the early developmental. When a child has difficulty acquiring and or using language appropriately. It can result in behavioral difficulties and negatively impact later academic achievement. language disorder for little kids specific learning disorder..for kids in school..learning age

WK3---SUN--->---36yo F presents w/ worsening pelvic pressure over the last yr...dull aching pain...pain/ deep penetrating intercourse..bmi 26..PE reveals enlarged midline pelvic mass that extends to umbilicus...mass is mobile..several palpable anterior protuberances..neg preg test -likely dx?

-leiomyoamata uteri/fibroids EO: benign uterine tumors common in reporductive age women..extremely large fibroids cause irregular uterine contour and bulk sxs such as pelvic pressure, deep pelvic pain w/ intercourse

WK3---THURS--->--62yo F presents w/ anxious, diaphoretic, tremulous , anxious and tachycardia....pmh of depression and chronic hepatitis dt hep c -most appropriate pharmacotherapy?

-lorazepam impatience with liver disease, benzodiazepines lorazepam oxazepam temazepam are preferred due to the short half-life and lack of active metabolize. Benzodiazepines with longer have lives in active metabolites, such as Florida except CHLORDIAZEPOXIDE and diazepam, please patients we have petty dysfunction at risk for toxic drug buildup.

WK3---WED--->---32yo F returns for followup after starting antidepressant 2wks ago..pt says she's concerned the medication my stop working since she's been worried about her dad's recent hospitalization and her husband is away for work. -most appropriate pharm mgt?

-maintain current dose of escitalopram Antidepressant trial is at least 4 to 6 weeks and a therapeutic dose. Physician should continue antidepressants for at least 4 to 6 weeks before considering the next step in treatment.

WK3---THURS--->--25 yo F presents w/ blasting..abd pain...fatigue..mood swings..HA...weight gain..dec lbibdo...episode have occur for yrs.. -nbsim

-menstrual diet.. - Premenstrual syndrome and premenstrual dysphoric disorder are characterized by physical such as fatigue bloating breast tenderness and psychological such as mood swings irritability symptoms that occur in the week prior to menses and resolved during the follicular phase. Assessment should begin with a menstrual diary to determine the relationship of symptoms to menstrual cycle phase.

WK3---THURS--->--32 yo M presents w/ complaints of daytime sleepiness for the past several months—worried that he will lose his job...PE is remarkable for obesity..he hear voices when falling asleep...most appropriate treatment?

-modafinil - The patient excessive daytime sleepiness HYPNAGOGIC hallucinations on falling asleep and sleep paralysis are suggestive of narcolepsy cataplexy emotional triggers loss of muscle time can also be seen. I am 0DAFINIL a non-amphetamine medication that promote wakefulness is considered first line pharmacotherapy as it reduces daytime sleepiness is generally well tolerated and has less abuse potential compare with traditional stimulants such as amphetamines and methylphenidate. Narcolepsy is characterized by excessive daytime sleepiness cataplexy HYPNAGOGIC/HYPNOPOMPIChallucinations and sleep paralysis. Treatment includes sleep hygiene schedule naps and avoidance of alcohol and drugs that cause drowsiness when medication I need it to decrease daytime somnolence some wakefulness promoting agents such as MODHFINILR preferred.

WK3---THURS--->---40 yo male present w/ complaints of BL hand tremor that he noticed 2months ago...worsens w/ anxiety..6months ago pt was treated for a manic episode.. -likely cause?

-medication induce adverse effect - Trimmer is common adverse effect of lithium, possibly due to increased iron accumulation in the substantia nigra. Lithium induced enhanced physiologic tremor is typically symmetric, limited to hands and upper limbs, and occurs when the medication is started or the dose is increased such as after a manic episode. It is non-progressive and often decreases over time even with no dosage reduction. Lithium enhanced physiologic tremor must be distinguished from tremor due to lithium toxicity, which typically causes any regular course tremor involving multiple parts of the body not just a hand accompanied by gastrointestinal Worden's no neurologic symptoms. The team toxicity requires hydration and hemodialysis in contrast to let them in Hanst physiologic trimmer is off he managed with watchful waiting. This tonic trimmers may present as a regular trimmers exacerbated by stress. They occur with sustained muscle contractions such as Raspas Simpson, typical in the same area as the trimmer. Next paragraph I sent your trimmer is also leucine most prominently in the hands and can be worsened by anxiety or outstretched hands. However it is predominantly an action trimmer so it becomes more pronounced at the end of goal directed movement and improves with rest. Essential tremor classically improves with alcohol. In additional patients are typically older or have a family history essential tremor is autosomal dominant. Hypothyroidism can cause an enhanced physiologic tremor indistinguishable from this patient based on trauma characteristics. However additional symptoms such as weight loss heat in tolerance and findings of hypothyroidism such as tachycardia hypertension hyper reflexive are typically present. Patient with Parkinson's disease classically have a resting tremor that improves with intentional movement. The trimmer is accompanied by rigidity bradykinesia in gait disturbances.

WK3---THURS--->--46 yo M present to office for f/u regarding htn....mood seems really low...he says he's been stressed at work dt people being laid off that has increased his work load..he reports difficulty falling asleep and his wife complains that he doesn't have time for her...on weekends he still enjoy socializing w/ friends -likely dx?

-normal stress respone bc pt is still able to go to work and enjoy socializing..he's not functionally impaired - Patients with situational triggered depression symptoms should be assessed for major depressive and adjustment disorders. Normal stress reactions are distinguished by lower severity and absence of market distress or significant functional impairment.

WK3---THURS--->--42 yo M presents w/sxs of depression since his divorce 3yrs ago...pmh of separation anxiety as a kid...and family hx of GAD.. -indications for longterm maintenance therapy...

-number of prior depressive episodes -MDD is frequently recurrent...LT maintence therapy antidepressant therapy is indicated for pt's w/ 2 or more MDD episodes. Other indications for LT maintenance therapy include early age onset..greater severity, suicidality...and presence of severe, ongoing psychosocial stressors or persistent residual depressive sxs

WK3---WED--->---15yo girl brought to the office by her mother bc of behavior..stays out past curfew...argues and slams door in mom's face...failing multiple classes at school...sent to principal office after refusing to put away phone in school.. -likely dx?

-oppositional defiant disorder - The patient's pattern of defiance of rules and authority figures, irritable mood, and blaming failures on others in suggestive of oppositional defiant disorder. For a diagnosis of ODD, the oppositional behaviors must be excessive compared to normative, age-appropriate behaviors and observed during interactions with individuals other than siblings. Opposite notify disorder is characterized by a pervasive pattern of argument give define behavior toward authority figures. It does not involve the more severe violations of basic rights of others as seen in conduct disorder.

WK3---WED--->--32yo M presents w/ concerns about recurrent episodes of sudden heart pounding..SOB...chest pain...dizziness...shaking...tingling in his extremities over the past several months..pt fear these episodes will happen at work -likely dx?

-panic disorder The unexpected and abrupt onset of symptoms in this otherwise healthy patient with repeated negative in medical work up a suggestive of panic disorder. Patient with panic disorder frequently have cardio respiratory symptoms and a fever they are having a heart attack and repeatedly seek medical care. The symptoms may include gastrointestinal distress, procedures, and dissociative symptoms such as the realization in the personalization. Next paragraph patient with panic disorder may be miss diagnosed with somatic symptom disorder due to preoccupation with unexplained symptoms and a history of high healthcare use. The recurrent abrupt onset of characteristic physical symptoms that resolve within minutes to raise clinical suspicion for panic disorder. And illness anxiety disorder patients experience excessive anxiety about having or acquiring a serious illness and have few or no somatic symptoms.

WK3---SAT--->---29 yo F presents w/ complaints of worsening of cystitis..6months ago pt had frequent suprapubic pain when bladder get full...urinary frequency...and suprapubic pain relieve w/ voiding...now pt says for the past week says the she's had inc urgency along with dysuria(pain w/ urinating)...PE and labs reveal pain on palpation of lower abd..UA shows positive for leukocyte esterase...WBC 40/hpf....urine culture and prep test is neg -nbsim?

-perform chlamydia and gonorrhea nucleic amplification testing -pain on urination is atypical for cystitis...the combination of dysuria and sterile pyuria is a common presentation for chlamydia...assoc w/ urethritis... EO: the combination of dysuria and sterile pyuria is a common presentation for chlamydia urethritis in sexually active women.Pts w/ this clinical presentation requires nucleic acid amplification testing for chlamydia.

WK3---SUN--->---Intrahepatic cholestasis of pregnancy (ICP) features—common in 3rd trimester

-presents w/ pruritus in the absence of a rash...elevated serum bile acid concentration....usually develops in 3rd trimester and resolves w/ delivery....pruritus on palms and sole that is worse at night...dt estrogen/progesterone over saturating the hepatic biliary system.... -elevated bile acid..cholic acid..and chenodeoxycholic acid confirms the diagnosis ICP carries inc risk of intrauterine fetal. Death...tx w/ ursodeoxycholic acid and induction of labor at term

WK3---WED--->--5yo boy is brought in by parents dt fear of thunderstorms...it has disrupted his sleep schedule and school attendance...when boy notices storm clouds he starts to cry and hide under his bed...if it happens at school he asks to go home -appropriate recommendation?

-read stories that depict storms w/ the patient Specific phobia is characterized by intense fear of an object or situation that is avoided when possible and causes significant impairment. The treatment of choice is gradual exposure to the feared object or situation.

WK3---SUN--->---38 yo F g3p3 presents w/ abnormal uterine bleeding for the past 6months...been having amennorheioc for past 3 months..w/ intermittent spotting..also notice abd bloating...coarse hair on upper lip..bitemporal hair thinning...large pelvic mass extending through LLQ.. -likely dx?

-sertoli-leydig cell tumor -the bitemporla hair thinning and large pelvic mass is dt the tumor...the sertoli and leydig cells are commonly found in testes but can develop in ovaries and produce testosterone..cell proliferation and inc test can lead to signs of virilization such as hair thinning...deep voice..inc muscle masss..clitomegaly...bc elevated test suppress pituitary FSH and LH release...pt develop sxs of anovulation and estrogen def such as abnormal uterine bleeding an amenorrhea EO=SL tumors are test secreting sex cord stroll tumors..presenting w/ rapid virilizations(months), amenorrhea and large pelvic mass PCOS could not have been the dx bc..its a progressive dz that happens over years(not in months) w/ slightly enlarged ovaries instead of one large mass in this .pt

WK3---WED--->--for the past year a 17yo girl has had persistent disturbing thoughts about stabbing her mom in the neck ..she constantly counts down from 5 to 1 to get rid of the thoughts.. -most appropriate management?

-sertraline - First line treatment for OCD includes exposure and response prevention a type of cognitive behavior therapy and pharmacotherapy with a certain nourishing antidepressant all SSRI antidepressants such as fluoxetine sertraline paroxetine fluvoxamine citalopram escitalopram and the TCAs are affective. SSRI are preferred as initial treatment due to superior in Perry's affect profile. Treatment of OCD consist of SSRI serotonergic antidepressants and CBT based on exposure and response prevention. SSRIs first my medication.

WK3---SUN--->---features of acute fatty liver of pregancy/pre-eclampsia(AFLP)—3rd-trimester issues!!

-severe cases cause elevated LFTs..acute RUQ pain..assoc w/ thrombocytopenia..results from micro vesicular fatty infiltration of hepatocytes...nausea...vom..abd pain...malaise..anorexia..jaundice...half of .pts get experience pre-eclampisa during course of illness..elevated ast..elvated..alt..elevated bilirubin..wbc may be elevated..platelets occasionally dec..sever cases show high serum ammonium..prolonged prothrombin and hypoglycemia from hepatic insufficiency...acute kidney injury and hyperuricemia often occur together

WK3---SAT--->----acute cervicitis

=presents as 1st trimester bleeding..PE shows closed friable cervix w/ mucopurulent discharge.

WK3---SAT--->----ectopic prep

=presents w/ unilateral pain and light vag bleeding

WK3---SAT--->----cervical insuff

=refers to painless cervical dilation during second trimester that results in loss of pregnancy

WK3---WED--->--24yo M brought to ER by wife bc of strange behavior...since pt has been fasting for the past 5 days...he's been snappy stopped going to work...current meds=acetaminophen and omeprazole...decreased sensation in finer tips and mild weakness in arms..along with constipation and abdominal pain..and pt says he's being hunted by large shadowy figures -likely dx?

Acute intermittent porphyria...w/ e/o abd pain psychiatric and neurologic abnormalities after following.. Acute intermittent porphyria is an autosomal dominant disorder involving alterations in enzymes involved in heme biosynthesis is characterized by acute intermittent neurovisceral attacks. New onset of psychiatric and neurologic abnormalities following a period of fasting, accompanied by unexplained, acute abdominal pain and family history of similar episode strongly suggestive of AIP. Elevated urinary PORHOBILINOGEN levels during an attack or diagnostic. Common precipitants of attacks including fasting such as carbohydrate restriction induces enzyme synthesis, certain medication such as carbamazepine, PHENYTOIN, rifampicin RIFAMPIN, tobacco, Alcohol neuropathic presentation include abdominal tenderness vomiting constipation sensory and motor neuropathy's such as decrease sensation in fingertips, and tachycardia psychiatric symptoms may be non-specific and include anxiety, Insomnia, restlessness, mood fluctuations, and psychotic symptoms such as hallucinations or delusions. AIP should be ex suspected in patients with sudden onset abdominal pain, neuropsychiatric symptoms

WK3---THURS--->--features of global developmental delay

Definition: significant delay in≥ 2of the major developmental domains (gross motor, fine motor, language, cognition, and social milestones) in children < 5 years of age EtiologyChromosomal abnormalitiesMetabolic disorders (e.g., phenylketonuria, lysosomal storage diseases, maple syrup urine disease)Infections (e.g., TORCH, herpes simplex, meningitis)Intrauterine exposure to toxic substances/drugs (e.g., alcohol, phenytoin, valproate)Birth complications (e.g., preeclampsia, multiparity)Untreated neonatal jaundice Clinical featuresCommonly associated with behavioral abnormalities (e.g., ADHD, autism spectrum disorders, bipolar disorders)

WK3---THURS--->--motivational interviewing components

Evoking change talk is a technique used in motivational interviewing a patient with substance use disorder. The patient is asked to assign a numeric rating to the level of motivation to change this is followed by open-ended questions to explore the patient's reasons for wanting to make a change. This process can help solidify the patient's motivation and prepare for the planning process.

WK3---SUN--->---requirements for preterm labor less than 32wks?

Indomethacin—>to inhibit contractions and delay delivery(tocolysis) -betamethasone to promote lung maturity -magnesium sulfide to dec the risk of cerebral palsy(neuro-protectant) -penicillin to dec risk of neonatal group B strep

WK3---SAT--->---31 yo F at 7wks GA presents to the ED w/ vaginal bleeding..saturated 3pad w/in 4hrs...along w/ lower abd pain...PE reveals dilated cervix and blood clots..and uterine size of 6wks..U/S shows gestational sac in lower part of uterus -like dx?

Inevitable abortion w/ eo vag bleeding...lower abd pain...dilated cervix w/out POC...U/S showed nonviable gestation in the lower uterine segment —threatened abortion=presents w/ bleeding ..closed cervix..and US shows intrauterine gestation w/ normal fetal heartbeat. -cervical insufficiency =refers to painless cervical dilation during 2nd trimester that results in loss of pregnancy

WK3---THURS--->--26yo F present w/ complaints about how her breast looks terrible and how she has been trying to hide the flaws...and stopped seeing friends...she is requesting plastic surgery... -most appropriate initial response?

It seems you would like to get back to doing things that you previously enjoyed instead of thinking about your appearance. Body dysmorphic disorder involves excessive preoccupation with perceived appearance flaws and is often accompanied by poor insight and a request for surgical procedures. The initial approach should focus on the suffering and impaired functioning without challenging the patient's relief about appearance. What rapport is it tablet, the patient should be educated about BDT and its treatment.

WK3---THURS--->--Bromocriptine

Medication for Parkinson disease → Overview An ergot derivative dopamine D2 receptor agonist. Indications include hyperprolactinemia, Parkinson disease, and neuroleptic malignant syndrome. Adverse effects include nausea and impulse control disorder.

WK3---THURS--->--Schizoaffective disorder

Schizophrenia → Differential diagnoses A psychiatric disorder characterized by the presence of psychosis (hallucinations, delusions, and ego disturbances), intermittently accompanied by manic or depressive symptoms. By definition, the psychotic symptoms must be more dominant than the mood symptoms and psychosis must have been present for at least 2 weeks in the absence of any mood disturbance

WK3---SUN--->----endometrial polyps—

Small growths of endometrial glands and storm w/in the uterine cavity..classically have intermenstrual spotting

WK3---SUN--->----42 yo F g2p2 presents w/ complaints of pelvic pain and heavy bleeding that requires changing tampon every hour...PE shows a boggy uterus thats tender to palpation -most likely present in this pt?

Symmetrically enlarged uterus Pts features points toward adenomyosis...common in multiparous women over 40...dysmenorrhea...heavy bleeding...chronic pelvic pain...buggy tender and symmetrically enlarged uterus adenomyosios typically present in women over 40 and characterized by dysmenorrhea..heavy menstrual bleeding..progressive chronic pelvic pain...and a boggy tender..symetrically enlarged uterus

WK3---FRI--->--meta-analysis

a quantitative statistical technique used to combine and analyze data from several studies to conduct an analysis with a greater statsitical power than that of the individual syudies

WK3---TUE--->-scarlet fever

erythrogenic= means inflammation and red skin..

WK3---SUN--->---16yo girl present LLQ pain that suddenly started overnight...pain rated 5/10...is sexually active but uses condoms....normal vitals...neg pregnancy test..PE shows anteverted uterus w/ tenderness over a slightly enlarged left adnexa...pelvis US shows left ovary w/ 3cm cyst w/ several subcentimeter cysts in the periphery(corpus luteum described)...normal doppler blood flow w/ small amt of free fluid in posterior-cut-de-sac(normal) -likely dx?

mittelschmerz The pts neg preg test and US that showed simple ovarian cyst w/ small amount of fluid in pelvis suggest benign etiology...also cycle was 2 wks ago which means she's ovulating now..and feeling the pain of the ruptured follicle EO:Mittelschmerz is a physiologic cause of unilateral abd pain n young women...the pain occurs in the middle of menstrual cycle(follicular phase)..corresponding w/ ovulation

WWK3---FRI--->--confidence interval

not stat significant if it includes the null such as OR=1

WK3---TUE--->-11yr old boy presents w/ irritability...declining grades...fidgety..change in sleep and appetite..anhedonia -likely dx -depression

Pediatric major depression may present with irritability rather than depressed mood. If a patient displays irritability along with social withdrawn and academic decline, major depression should be considered.

WK3---TUE--->--8yo girl present w/ lethargy...cough..fever..HA...for past 3 wks..went on a recent trip to Ecuador...tmp is 103.6..dec breath sound and crackles in lobe...css reveals low glucose...elevated protein w/ 70% lymphocytes... -likely cause?

TB meningitis is rare but serious..dt hematogenous spread of mycobacteria to subarachnoid space..onset is gradual w/ weeks of non specific sxs...bf progression to fulminant meningitis...CSF analysis shows... lymphocytosis..high protein..and low glucose

WK3---MON--->-3yo boy w/ sickle cell anemia brought to the ER dt fever....bp 78/40...RR22...lethargic w/ pallor...leukocytes 18KL..neutrophils 80% -likely organism?>

The patient's fever chills hypertension leukocytosis and bandemia are consistent with sepsis. Intermittent cycling of the red blood cells in the spleen pleased to clinic infarction and ultimately functional asplenia. As a result the patient with sickle cell disease are at high risk for sepsis from strep pneumonia, H influenza, neisseria. Sickle cell disease causes functional asplenia due to recurrent clinic infarction. Therefore, patients are at risk of overwhelming interaction with encapsulated organisms and should receive vaccination and penicillin prophylaxis. Pneumococcus remains the most common cause of sepsis patients with sickle cell disease.

WK3---MON--->-4yo bo presents w/ rash...round spots on left foot..itchy...and painful...some lesions are covered w/ a transparent yellow crustin -nbsim?

Topical mupirocin..pt has e/o impetigo...dt yellow crusting Non-bullous in the tiger is characterized by populace and pustules with overlying honey colored crusting. Topical antibiotics such as mupirocin are the treatment of choice for localized infection.

WK3---TUE--->--features of trachoma

Trachoma Conjunctivitis → Bacterial conjunctivitis A condition of chronic keratoconjunctivitis caused by recurrent Chlamydia trachomatis infection. Infection results in an active phase of conjunctivitis and a cicatricial phase characterized by conjunctival scarring.

WK3---MON--->-3yo boy present to ER bc of limping for past 3 days after mom gets him from daycare..pt refuses to stand...2months ago had fever and runny nose..today he also has fever and tachypnea..lymphadenopathy..hepatomegaly..during movement of left hip..opt wails..cbc show leukocyte of 15K w/ 80% neutrophils Nbsim?

bilateral hip ultrasound acute atraumatic hip pain in children is typically caused by transient synovitis, which present with the well appearing children who are often afebrile and able to ambulate. However, patients with features concerning for septic arthritis such as inability to ambulate, leukocytosis required bilateral hip ultrasound with or without arthrocentesis to distinguish between the conditions. Ibuprofen includes follow up are the management of transient synovitis but since this patient's inability to ambulate requires further evaluation.

WK3---MON--->-2yo girl presents w/ lethargy and 1 day of confusion...has a URI 4 days ago...pupils are equal and sluggish to light..labs show ec bicarb...elevated ALT and AST...elevated ammonia -help establish dx? -medication hx

-Reye syndrome present in young children with rapidly progressive encephalopathy and hepatic dysfunction after aspirin administration for a viral infection. Characteristic laboratory findings include elevated aminotransferases and hyperammonia. Coagulopathy, hypoglycemia, and metabolic acidosis are also common. -Cholinergic toxicity such as organophosphate exposure presents with vomiting, diarrhea, meiosis, and bradycardia, as well as excess urination, tearing, and salvation.

WK3---MON--->-13yo girl presents w/ frequent headaches that awaken her from sleep....height in 5th percentile...weight in 75th percentile...PE reveal optic disc edema...breast tanner 1 MRI would show?

-suprasellar massw/ calcifications..w/ e/o craniopharygiomas - Craniopharyngiomas are benign slow-growing calcified tumors but then the suprasellar region. Tumor compression of the optic chiasm can result in temporal hemianopsia and pituitary stock compression can cause panhypopituitarism such as growth failure and pubertal delay Acoustic neuroma is a benign tumor of the vestibular nerve. On imaging it appears as enhancing lesion in the internal auditory canal patients are usually adults with hearing loss/tinnitus and mild ataxia due to vestibular dysfunction Brain abscess appears as an intraparenchymal lesion with air fluid levels and a hyperdense rim on imaging. Patients typically have an acute headache, focal neurologic findings such as sensory motor deficits, and often fever Patient's tuberous sclerosis --build up in the lateral ventricles these nodules can increase intracranial pressure due to obstruction struct of hydrocephalus. Patient presents with ash leaf spots and angiofibroma Medulloblastoma arises from the cerebellar vermis and presents with vomiting, headache, cerebellar dysfunction such as ataxia dysmetria.

WK3---MON--->-3yo boy swallows a battery..seen on chest X-ray Nbsim?

A button battery lodged in the esophagus can cause tissue corrosion, necrosis, and life-threatening perforation. A circular object with the halo sign on chest x-ray is consistent with a button battery, and management is immediate endoscopy removal.

WK3---MON--->-3yo boy presents to the ED w/ persistent cough..pleuritic chest pain..and SOB...4days ago he was prescribed antibiotics but seems to be getting worse..temp tis 102.6..bp=80/40...Osat=91%..BL crackles..CXR show right middle lobe,RLL and LLL infiltrates..inaddition to changing amoxicillin to ceftriaxone..nbsim?

Add IV vancomycin therapy.. In children, the most common pathogens that cause in Pa'ina or strep pneumonia and staff aureus, including methicillin resistant staph aureus. Empiric treatment is with ceftriaxone and Vanco myosin to target against these pathogens. The book says also add chest tube

WK3---MON--->-approach to dehydration

Approach Initial fluids for dehydration and hypovolemia: Administer isotonic fluid and choose route and rate based on estimated fluid loss.Severe fluid loss (hypovolemic shock): Begin immediate hemodynamic support with aggressive IV fluid resuscitation, e.g., 20 mL/kg bolus of isotonic crystalloid.Moderate fluid loss (hypovolemia without shock)Administer enteral OR parenteral fluids to correct extracellular volume deficit.If starting with IV fluids, transition to enteral fluids as soon as possible.Mild fluid loss: Prioritize enteral replacement of fluids (i.e., oral rehydration therapy). Supportive care for all patients: Management of these is often begun concurrently with initial fluid administration.Treat associated metabolic disturbances: e.g., glucose and electrolyte abnormalities.Identify and treat underlying causes (see "Etiology").Address continued fluid needs.Replace free water deficit to restore intracellular volume losses and correct hypernatremia.Meet maintenance fluid requirements.Replace ongoing fluid losses. Stabilization through correction of intravascular volume deficit with fluid resuscitation is the first priority. Manage urgent metabolic abnormalities (e.g., severe symptomatic hyponatremia, acute hypoglycemia) concurrently with fluid resuscitation. Address subacute electrolyte abnormalities after stabilization.

WK3---TUE--->-What is the most likely underlying cause of this patient's condition of primary ammenorhea?

Congenital absence of an X chromosome Patients with turner syndrome have primary amenorrhea and lack of breast development due to low estrogen levels associated with gonadal dysgenesis. Lack of feedback inhibition by estrogen on the hypothalamus pituitary ovarian access results in increased FSH & LH. An absent X chromosome on karyotype analysis is diagnostic. The malarian and duct develops into the uterus, fallopian tubes, and upper 2/3 of the vagina. Patient with abnormal development typically present with amenorrhea however uterine anomalies are seen on ultrasound. Ovarian development is not affected therefore FSH & LH levels are normal. Patients with androgen insensitivity syndrome can have a Minorea however they typically have tall stature and breast development due to androgen conversion to estrogen. The uterus is not seen on ultrasound. Patient with congenital GNRH deficiency such as Kallmann syndrome can have amMenorea due to decreased hypokalemic stimulation of FSH & LH Excess endogenous glucocorticoids such as Cushing syndrome suppress GnRH, causing low FSH & LH levels Functional hypothalamic amenorrhea occurs in adolescent girls with severe calorie restriction such as athletes or anorexia nervosa due to suppression of their hPO axis. FSH and LH levels are low

WK3---WED--->--32 yo F has been feeling anxious...having nightmares...episodes of SOB..palpitations and sweating for the past 2wks after getting robbed while walking home from work.. -likely dx?

- - The patient who recently experienced trauma the mugging event as a two week history of anxiety impaired concentration insomnia dissociative symptoms such as being in a daze and avoidance of discussing the Trumatic event, consistent with acute stress disorder. Her negative mood and tearful knees are also characteristic. Other ASD symptoms include memories, nightmares, and flashbacks. Symptoms symptoms must last for three days or more but less than a month to meet the following trauma exposure criteria. Acute stress disorder is a severe anxiety response characterized by Rí experience Inc. of trauma, dissociation, negative mood, avoidance and hyper arousal lasting from three days to a month after exposure to a traumatic event. If the symptoms last longer than one month then it meets the criteria for PTSD. ASD and PTSD should be treated first line therapy trauma focused cognitive behavioral therapy. Depersonalization/D realization disorder is characterized by a sense of unreality of self or environment. Generalized anxiety disorder also have insomnia, tension, and difficulty concentrating. Anxiety and generalized anxiety is about multiple every day concerns rather than being trauma related and has a duration of six months or longer.

WK3---THURS--->--26yo F brought to ED by coworkers..she's agitated..aggressives...strike out at nurse...nystagmus -explantion -?Phencyclidine

- PCP is a disassociative anesthetic that characteristically causes nystagmus. It can also cause associated feelings psychotic and violent behavior, severe hypertension, and hyperthermia. -tx w/ benzodiazepines

WK3---WED--->--72 yo F present for f/u ...currently in remission after battling breast cancer...despite good news...pt's daughter says she eats little...not happy....losing weight...difficulty concentrating...and remembering..pt says wouldn't anyone who've been through this is depressed? -nbsim?

- Recommend antidepressant therapy and psychotherapy The patient's history of depressed mood, anhedonia in ability to enjoy a normal pleasurable activities, difficulty sleeping, appetite disturbance, love energy, and impaired concentration meet the criteria for major depressive disorder. The SIGECAP yes mnemonic can assist and recall of the course signs and symptoms of major depressive disorder. A diagnosis must have 5 of the 9 sxs for at least two weeks; one of the symptoms must be anhedonia or depressed mood. Although some depressive symptoms in cancer patients are common, the development of MDD should not be normalized and considered as an expected reaction. Untreated depression in medically ill patients can reduce the quality of life and additional Lee english for families and can negatively impact treatment adherence in medical management. Medically ill patients who develop comorbid depression can benefit from treatment with antidepressant medication and psychotherapy to improve their quality of life.

WK3---MON--->--5day old girl present w/fussiness..non bloody non bilious vomiting after feeds for the past day..bilateral cataracts hepatomegaly -cause of this newborn's presentation?

-newborn inability to metabolize breast milk.. Galactosemia is an inborn errors of metabolism that present in the first week of life with vomiting, jaundice, hypo Tonea, and hepatomegaly. Progressive deposition uncle Lassiter, byproduct of collectors, leads to Cataracs management is a galactose Dash free diet such as soy based formula.

WK3---TUE--->--6month old boy cant sit w/out support...head is large for his age..vomits frequently at the end of sleep..mri show a mass in the left ventricle -likely cause? -increase CSF production

-- Choroid plexus papilloma is a benign intraventricular mass that causes increased production of cerebrospinal fluid, leading to ventriculomegaly and hydrocephalus. Infants, hydrocephalus presents with enlarging head circumference and signs of increased intracranial pressure. Infants, accumulation of a lot of CSF such as hydrocephalus causes in large hair circumference in size increase intracranial pressure such as for feeding, bulging fontanelle, irritability vomiting on waking developmental delays such as inability to sit with support at six months. Choroid plexus papilloma Hydrocephalus → Etiology and pathophysiology A rare benign tumor of the choroid plexus. Typically presents with headache (and other symptoms of intracranial hypertension) due to hydrocephalus secondary to cerebrospinal fluid overproduction. -Congenital atresia of the Farahmand of Luske imagine deeper Vince CSF outflow into the subarachnoid space causing hydrocephalus. This congenital anomaly is associated with dandy walker syndrome. Increase pressure in the vina sinus is due to dural sign Istanbul sis can impair see SF absorption and lead to hydrocephalus. Most patients but Enduro sinus thrombosis are adults with prothrombotic risk factors such as pregnancy and anti-thrombin deficiency.

WK3---MON--->-1day old baby presents w/ oxygen sat of 99% in right hand and 91% in right foot.. -likely cause of pt's clinical presentation?

--persistence of fetal circulation persistent pulmonary hypertension of the newborn is characterized by abnormal persistence of elevated pulmonary vascular resistance, causing right to left Shanti across the patent doctors arteriosus. Patients have a low post doctoral saturation and normal distal pulses. Right to left shunt thing across the ductus arteriosus also occurs in conditions in which left ventricular output is obstructed such as coarction the aorta. These infants have diminished tomorrow pulse pulses and often lower extremity pallor due to decreased distal blood flow. Left to right intracardiac shunt such as a true septal defect and ventricular septal defect calls oxygenated blood to return to the right side circulation before being pumped to the lungs again. Total anomalous pulmonary venous return causes oxygenated blood to return to the right side of the heart; affected Evans have a right to laugh intracardiac shunt to provide systemic blood flow. Similarly, conditions with right ventricular outflow tract obstruction such as tricuspid atresia and tetralogy of ballot require right to left intracardiac shunt in through a septa defect. These infants are often cyanotic and do not have a gradient between pre-and postductal saturation because all mixing is intracardiac, occurring before the ductus arteriosus. -Persistent pulmonary hypertension of the newborn Neonatal respiratory distress syndrome → Differential diagnoses Persistent pulmonary hypertension Abbreviation: PPHN A condition in which the pulmonary vascular resistance remains abnormally high after birth. This results in right-to-left shunting of blood via the fetal circulatory pathways.

WK3---MON--->-10-hour old boy presents w/ bilious emesis...hypotonia..flat face...prominent tongue..low set ears..and slanted palpebral fissures...holosystolic murmur detected at LLSB w/ precordial thrill..ABD X-ray reveals double bubble -NBSIM?

-0surgical repair bc duodenal atresia needs to be fixed immediately Duodenal atresia associated with down syndrome and presents with Bilious vomiting in the first two days of life and a double bubble on abdominal x-ray. Treatment is surgical repair Sweat testing for cystic fibrosis is necessary for any neonate with meconium ileus. Meconium ileus causes delayed passage of meconium, marked abdominal distention and sometimes Bilious emesis. X-ray shows dilated loops of small bowel due to terminal ileum obstruction. Upper G.I. series is the gold standard for diagnosing malrotation, which also cause bilious emesis,malrotation presents with abdominal distention and distal intestinal gas on x-ray.

WK3---MON--->--5yo girl presents w/ leg pain...worse at night for past 2 days...refusing to walk...temp os 100.9...hepatomegaly..pale..scattered bruised across chest and back..palpation od anterior tibia elicits pain...labs show anemia and thrombocytopenia.. -casue of pt sis?

-Acute lymphoblastic leukemia w. E/o bone pain..panycytopenia and age of pt Bone pain in pancytopenia in a child are concerning for acute lymphoblastic leukemia. Other characteristic findings include fever, pallor, bruising, and hepatosplenomegaly. bone marrow failure due to a plastic anemia is characterized by pancytopenia. The hypocellular bone marrow of a plastic anemia does not typically cause bone pain ITP is characterized by bruising and petechiae due to low platelet count. However hemoglobin and white blood cell count would be normal. Juvenile idiopathic arthritis is characterized by joint pain, Swelling, and morning stiffness.

WK3---MON--->--5yo girl w/ joint pain and rash...last week had pain in knees but now has pain in ankles an wrists are tender...developed nonpruritic pink rash on back...temp 101...had sore throat a few weeks ago..PE reveals multiple large well demarcated red non itchy patches w/ slightly rain outline on trunk and limbs...labs showed elevated C-reactive protein and elevated ESR -likely dx?

-Acute rheumatic fever w/ e/o joint pain..recent pharyngitis..nodules..erythema marginatum fever arthalgias.. Acute rheumatic fever, a complication of streptococcal pharyngitis, is diagnosed clinically using the Jones criteria which includes fever migratory poly arthritis erythema marginata and elevated acute phase reactant. Systemic juvenile idiopathic arthritis is diagnosed when arthritis is present for more than six weeks. Systemic symptoms include fever and rash, it can be seen in children with systemic juvenile idiopathic arthritis but the arthritis is usually not migratory. Systemic lupus erythematosus is an autoimmune disease that affects multiple organ systems and is rare in young children. Arthritis and elevated CRP and ESR can be seen, but rash is usually present across the cheeks such as malar rash Lyme disease is a tickborne illness that can present with rash and arthritis. The rash consists of single or multiple Bullseye lesions also known as erythema migrans that slowly expand. However in contrast to this patient, those with Lyme disease usually have monoarticular arthritis typically involving the knee. HSP is a vasculitis that can cause a transient migratory arthritis and rash. However, The rash is purpuric, and arthritis typically involves the lower extremity joints such as hip knee and ankle.

WK3---WED--->--19yo F is struggling academically and referred to the DR for anxiety..insomnia..fatigue...she worries excessively about her door being locked and stove turned off multiple times during day and night...she wont stop thinking about it until she checks it. - most appropriate treatment?

-CBT - The patient has obsessive compulsive disorder, Acadie a condition characterized by time-consuming recurrent unwanted thoughts/obsessions and repetitive behaviors that the patient feels compelled to perform/compulsion. First time treatment include cognitive behavior therapy based on exposure and response prevention and or pharmacotherapy with a selective serotonin reuptake inhibitor such as fluoxetine fluoxetine paroxetine sertraline. BUSPIRONE is used to treat generalized anxiety disorder. CLOMIPRAMI i NE is a tricyclic antidepressant that inhibit the reuptake of serotonin and norepinephrine, it is not used first line due to its unfavorable side effects such as sedation urinary hesitancy orthostatic hypotension and cardiac conduction delay. DIALECTICAL behavioral therapy is a type of psychotherapy developed for borderline personality disorder that integrates standard CBT techniques with principles of mindfulness, distress tolerance, and emotion regulation. Psychodynamic psychotherapy traces problems back to their origins in childhood and unconscious conflict may provide the patient with insight. Support psychotherapy is a broadly applicable approach that helps reduce stress and improve coping skills and maintain help.

WK3---TUE--->-21 month old presents w/ epiglottis...stridor...drooling..suprasternal and intercostal retractions..pt was intubated -best abx therapy to provide?

-Ceftriaxone and Vancomycin After securing the airway in patients with epiglottitis, initial treatment consist of broad-spectrum antibiotic therapy with ceftriaxone targeting H flu and strep species and vancomycin targeting staph aureus. Ampicillin and gentamicin is empiric therapy for neonatal sepsis which targets the most commonly identify pathogens including group B strep and E. coli Azithromycin is used for lower respiratory tract infections caused by atypical bacteria such as mycoplasma pneumonia, chlamydia pneumonia and for whooping cough caused by Bordetella pertussis Metronidazole can be used to treat CDF. Metronidazole also provides coverage against anaerobic organisms. Pipercillin-tazobctam and tobramycin are used to treat pseudomonas commonly seen in pulmonary infections and cystic fibrosis.

WK3---TUE--->--9yo girl presents w/ recurrent vomititng..no sxs today but has had nausea/vomiting and diffuse abd pain for several days over the last 8 months..usually begins in the morning and happens 10 to 15 times..happed 5 times in a 2 month span...its more frequent at the beginning of the school year..height and weight are normal -likely cause?

-Cyclic vomiting syndrome - This patient has a clear pattern of illness surrounding times of stress, as well as examination findings consistent with recurrent vomiting such as dental caries. In children, recurrent, self-limiting episodes of profuse vomiting and nausea without any apparent cause suggest cyclic vomiting syndrome. Key historical features include a history of identifiable triggers such as stress infection where a personal family history of migraines. -Bulimia nervosa is characterized by recurrent binge eating followed by inappropriate compensation behavior to prevent weight gain such as self-induced vomiting or purging. you'll also see a history of calorie restriction, Binge eating, or preoccupation with body weight and shape. -Chronic pancreatitis may present with recurrent bouts of abdominal pain, nausea, and vomiting. Poor growth and diarrhea are common, and examination reveals epigastric tenderness to palpation. -Factitious disorder imposed on another is a form of child abuse, and which a caregiver intentionally induces illness in a child or reports inconsistent symptoms. -Gerd in children is characterized by regurgitation or epigastric or substernal pain. -Inflammatory bowel disease typically presents as colitis, or abdominal pain, and diarrhea. Fever weight loss and abdominal tenderness on examination are common. -An idiopathic condition that affects both adults and children and is characterized by recurrent episodes of severe vomiting with normal health between episodes. In adults, the Rome III criteria for cyclical vomiting syndrome include a history of ≥ 3 episodes of acute vomiting per year, with each episode lasting < 1 week, and asymptomatic intervals between episodes.

WK3---TUE--->-1month old baby presents w/ blood-streaked stools..exclusively breastfed...past 3 days had loose stools streaked w/ blood...FOBT was positive -likely cause?

-Food protein-induced allergic proctocolitis Food protein induced allergic proctocolitis is a common benign reaction to proteins such as cows milk in breastmilk or farmer formula. Presentation involves a whale appearing in pain with painless bloody stools. Intussusception occurs in infants and toddlers age 6 to 36 months. He presents with lethargic, vomiting, And episodic crampy pain/irritability as one file segment telescopes in to another. Hematochezia is a sign indicates bowel ischemia. meckel diverticulum classically manifest in older children as painless hematochezia due to ectopic gastric tissue, bleeding in patients under six months is excessively rare and typically is associated with complications such as obstruction perforation intussususception. Your appearance and Iris ability due to pain would be typical mocked diverticulum particular cases. -Food protein-induced proctocolitis Inflammation of the distal colon caused by a non-IgE-mediated immune response against food protein antigens, most commonly from dairy or soy products. Typically presents as rectal bleeding in infants. Managed with dietary changes in the infant (if formula-fed) or the mother (if breastfed).

WK3---TUE--->-18yo F present to ED dt 3day of fever..HA...neck pain...temp 102.4..maculpapular rash on chest abdomen and extremities..CSF shows glucose=50...protein=90...lymphocytes=90%..likely organism?

-HIV w/ sxs of aseptic meningitis.. The patient's acute fever headache nuchal rigidity and photophobia raises suspicion for meningitis. Subsequent cerebral spinal fluid analysis reveals lymphocytosis mildly elevated protein in normal glucose indicating aseptic viral meningitis acute HIV infection often causes unexplained fever maculopapular rash, and aseptic meningitis. It should be suspected it sexually active or high-risk patients when more common causes of these symptoms have been excluded.

WK3---TUE--->--8yo boy w/ sickle cell dz..experiencing a pain crisis.. -expected to see on blood smear?

-Howell jolly bodies are remnant of red blood cells that are usually removed by a functional spleen. Due to functional asplenia and sickle cell patients, how jolly bodies are expected findings on smears. -Basophilic stippling represents ribosomal precipitates that appear as blue granules of various sizes disbursed throughout the cytoplasm of red less cells. It is seen with lead intoxication...sideroblastic anemia...thalassemia -Helmet cells are schistocytes or fragments of red blood cells. Their present suggest traumatic microangiopathic hemolytic condition such as DIC, hemolytic uremic syndrome, TTP....thrombocytopenia Purpera.

WK3---THURS--->---19yo F presents to the clinic dt cold intolerance...difficulty losing weight despite dieting and exercise..also complains of constipation...family hx gf graves...BP=80/50...pulse=38...BMI=15 -nbsim?

-admit pt to the hospitla - Indications for hospitalization in patients with anorexia nervosa include unstable vital signs, cardiac dysrhythmias electrolyte arrangements and severely low body weight. This patient is medically unstable and fulfills criteria for hospitalization her pulse is under 40 her blood pressure is under 80/60 she's hypokalemic hypophosphatemia wheels should be supervised some patients may require nasogastric feeding tube's.

WK3---WED--->--73 yo M presents w/ vomiting...confusion..ataxia...tremors since he was prescribed a new blood pressure medication while taking a prescribed mood stabilizer -which med caused his sxs?

-Lithiumn Thiazide diuretics can cause a decrease in renal clearance of lithium and lead to lithium toxicity. The risk of lithium toxicity is higher in patients with dehydration from any cause such as vomiting and diarrhea fever and renal dz and in elderly patients due to lower GFR and reduce the volume of distribution. Other medication's that can increase lithium levels include ace inhibitors, NSAID drugs, tetracyclines and metronidazole Concurrent use of thiazide diuretics as well as ace inhibitors, tetracycline, metronidazole, NSAID drugs, and lithium can increase serum lithium levels and lead to symptoms of lithium toxicity such as gastrointestinal symptoms, confusion, ataxia, tremor, and seizures. Bupropion NE/EPi nreuptake inhibitor —used as an antidepressant and assist w/ smoking cessation. Haloperidol and risperidone are dopamine antagonist used to treat psychosis or as adjunct to mood stabilizers and the treatment of bipolar disorder. Lamotrigine and valproic acid or anticonvulsants that are used in the treatment of bipolar disorder. Lamotrigine is associated with Stevens Johnson syndrome. Valproic acid -->gastrointestinal symptoms, hepatitis, pancreatitis, and hepatic encephalopathy.

WK3---FRI--->--research group evaluating the safety/efficacy bt quick penetrating solution of heparin(treatment) to heparin topical gel(control)....results shows the incidence of phlebitis is 60% w/ the QPS and 75% w/ the gel.. -most accurate interpretation of the these results -calculate and define the NNT

-NNT is 7 -NNT=1/ARR -ARR=control-treatment ARR=.75-.60=.15 NNT=1/0.15=6.66~7 ppl needed to be treated to prevent one negative outcome the NNT is the number of ppl who need to receive a treatment to prevent 1 additional negative event. NNT is the inverse of the absolute risk reduction. The lower the NNT, the more effective the treatment bc fewer ppl need to be treated to prevemt 1 additional negative event.

WK3---FRI--->--a new screening for anxiety is used...86%sens and 78% spec..on a smaple of 400students...whats the probability htat students in this sample w/ a negative test doesnt have the dz? -what concept is being tested?

-NPV is the probability that s/o does not have the dz ..given an neg test result NPV=TN/TN-FN

WK3---SUN--->---27 yo F presents w/ inquiries about contracepción...bleeds heavy doing cycles that required a tampon and pad...she and mom are heterozygous for factor V Leiden(clotting dz) and clotting dz run in family...CBC show Hb=10(anemic) -most appropriate contraception?

-Progestin releasing IUD EO:estrogen containing contraception methods are contraindicated own.pts at inc risk for thromboembolism...progestin -only contraceptives such as levonorgestrel releasing IUD and copper IUD may be offered...however copper IUD is assoc w/ heavy menstreual bleeding w/ anemia which could worsen the .pts sxs since she already has a low hemoglobin

WK3---THURS--->--serotonin syndrome features

-Serotonin syndrome [18] Description: a life-threatening condition caused by serotonergic overactivity Cause: ingestion of any drug that increases serotonin levelsMAOIs, SSRIs, SNRIs, TCAs, vortioxetine, vilazodone, trazodone, buspironetramadol, ondansetron, MDMA, dextromethorphan, meperidine, St. John's wort, triptans, linezolidIncreased risk withConcurrent use of two or more serotonergic drugsSwitching from one serotonergic drug to another without tapering Clinical features [18]Classic triad: neuromuscular excitability, autonomic dysfunction, altered mental statusdiaphoresis, hyperthermiahypertension, tachycardianausea; diarrheadelirium, psychomotor agitationhypertonia (especially in the lower extremities), hyperreflexia, myoclonus, tremor; mydriasis; seizure Treatment [18]Immediate discontinuation of serotonergic drugsSupportive careAntihypertensives, fluid replacementBenzodiazepines for sedationCyproheptadine5-HT2A receptor antagonists Serotonin syndrome causes HARM: Hyperthermia, Autonomic instability, Rise in blood pressure, and Myoclonus.

WK3---WED--->--31yo M presents w/ psychotic episode of schizophrenia...hearing voices...say halodol worked but had sxs of muscle spasms in neck...restlessness ...difficulty sitting still.. -most appropriate med?

-Ziprasidone EPS such as acute dystonia reaction occurred which likely contributed to the patient's nonadherence.. are common in first generation antipsychotics such as haloperidol due to their potency D2 antagonism in the Nigrostriatal ptw . The best approach in this patient would be to treat with a second generation antipsychotic such as Ziprasidone it reduces the risk of EpS. Among the second generation antipsychotics, the Ziprasidone has a low metabolic risk profile, whereas Olanzapine is associated with the highest risk of weight gain and metabolic side effects such as insulin resistant hyperlipidemia. Compare with first generation antipsychotic second-generation antipsychotics causes fewer EPS but are associated with metabolic affects to varying degrees. Clozapine and olanzapine carry a higher risk of metabolic side effects we are as zipprasidone or ariprprazolel and lurasidone associated with the lowest risk. -

WK3---THURS--->--22yo M present w/ complaints of difficulty sleeping and concentrating...lost his job 2months ago and had to move in w/ parents...he's easily distracted..eats more..gained weight..takwe 2 to 3 hrs to fall asleep... -most appropriate med?

-Zolpidem -pot clearly has adjustment disorder..developing w/in 3 months w/ identifiable stressor being job los nd moving back home...short term pharmacotherapy is the primary form of tx..ptS w/ AD experience pan excessive response to a stressful life event that impairs daily fun..including sleep...short term pharmacotherapy for insomnia can be used as an adjunct when more rapid symptom relief is desired

WK3---WED--->--22yo F brought to ER from a party by friends unarousable...bp 100/60...pulse 54...RR6...pupils 3mm(normal)..naloxone is given and nothing happens -which substances is the likely cause?

-alcohol and benzodiazepines This patient's presentation is consistent with sedated hypnotic overdose most likely due to the combined effects of alcohol and benzodiazepines. An isolated overdose of benzodiazepines can cause altered level of consciousness, ataxia, and slurred speech but is unlikely to cause this degree of central nervous system depression. Most patients with isolated benzodiazepine overdose or arousable and have normal vital signs. If signs such as bradycardia, hypotension, respiratory depression, and hyporeflexia are seen, as in this patient, congestion of sedated hypnotics should be suspected.

WK3---WED--->--48yr old man experienced auditory hallucinations 12hrs after being admitted to the hospital...he smokes and drinks daily for past 10yrs...hallucinations resolve by day2 in the hospital and his vital are normal -likely dx?

-alcoholic hallucinosis— Alcoholic hallucinosis is an alcohol withdrawal syndrome they typically present after 12 hours of abstinence and resolves within 48 hours after the last drink. Unlike delirium tremens, it tends to present a patient who are fully oriented and with relatively stable vital signs. The patient history of alcohol misuse, lack of pre-morbid psychosis, acute onset of auditory hallucinations shortly after hospitalization such as abrupt cessation of alcohol are most consistent with alcohol hallucinosis. Alcohol and alcohol withdrawal syndrome syndrome characterized by hallucinations and relatively stable vitals in an otherwise alert and oriented patient. Alcoholichallucinosis typically present after 12 hours of abstinence and resolved within 48 hours what is after the last drink this condition is usually self limited but can be managed with benzodiazepines. Cocaine withdrawal consist of depression, anxiety, fatigue, and intense cry events. PHENCYCLIDINEE intoxication can present with psychosis in addition to combative behavior, Delirium, dissociative symptoms, ataxia and nystagmus. Brief psychotic disorder is characterized by sudden onset psychotic symptoms lasting more than one day BUT less than a month. Is a rare disorder that can be diagnosed only after psychosis due to substance intoxication/withdrawal or medication condition is ruled out.

7yo M w/ painless patchy hair loss..narrowing at the shafts -likely cause?

-alopecie areata,..,. Alopecia areata is characterized by painless hair loss with no erythema scaling or scarring. It is an autoimmune disorder and is often associated with other autoimmune conditions such thyroid disease. Treatment includes topical or intra-lesional cortical steroids. A recurring course is common but most patients every growth over time. Telogen effluvium is characterized by diffuse rather than patchy hair loss.. It is often triggered by physiologic stressors such as severe illness pregnancy endocrine disorders or emotional distress. pediculosis capitis -- head lice present at the scalp..visible knits.

WK3---MON--->--11 yo boy presents to the ED w fever..productive cough...and worsening dyspnea..pmh of CF..temp is 101.3...110/min..RR 30/min..BL fine crackles and rhoncu..pt was on abs for a week..no improvement..Labs sho eosinophilia and elevated IgE.... -likely cause?

-bronchopulmonary aspergillosis w/ e/o CF..a week of abx w/ no improvement..eosiniphilia and elevated IgE Patients with structural airway disease, especially cystic fibrosis and asthma, are susceptible to allergic bronchopulmonary aspergillosis a BPA. Allergic sensitization to aspergillus spores is signified by high levels of circulating IGE. A BPA must be suspected when lung function declines without explanation, or persistent lower respiratory infection symptoms occur despite adequate antibiotic therapy. Wegners can cause constitutional symptoms of fever and malaise and necrotizing bronchiectasis, mimicking infectious pneumonia and responsive to antibiotic therapy it typically affects men over 60. Acute hypersensitivity pneumonitis can cause fever, malaise, and pneumonia like airspace infiltrates unresponsive to antibiotic therapy. Pneumonitis typically improves spontaneously in the hospital meaning the patient was removed from the inciting exposure Simple non-disseminated helmet infection can cause eosinophilic pulmonary disease such as infiltrates with peripheral eosinophilia and elevated IGE. However, it usually occurs in rural tropical and subtropical areas in addition because larva penetrate the skin in or eventually swallowed, patients usually also have dermatologic roaming urticaria and Gastrointestinal obstruction manifestations such as ileus or structure Patients with CF bronchiectasis can be colonized and affected by nontuberculous mycobacteria a nTM such as mycobacterium avian complex however in NTM disease is chronic such as months to years and presents with nodular, cavitary, and eventually fibrotic lung disease.

WK3---TUE--->--A 14 year old girl presents to the office because she has not had a menstrual cycle... she has no breast or development of axillary hair...ultrasound confirmed the presence of a uterus next best step in management?

-check fsh levels -Primary amenorrhea is the absence of menstrual cycle and girls 13 or older with no secondary sexual characteristics. And patients with a uterus, the best next step is and FSH level test which distinguishes between central low to normal FSH and peripheral amenorrhea

WK3---TUE--->--newborn baby born macrosomic..mom is diabetic...vitals and physiocal exam is normal but baby is jittery..glucoe is 65(hypoglycemia is 40).. -nbsim?

-check serum ca2+ levels The most common cause of journeys jitterynes in infants of diabetic mothers is hypoglycemia. However the patient serum glucose is now normal making hypoglycemia not the casue... The pathophysiology hypocalcemia in infants of diabetic moms relates to maternal hypo magnesium which is caused by osmotic diuresis in the setting of poorly controlled gestational diabetes. magnesium concentrations which reflects maternal levels lead to PTH suppression and low calcium levels in the neonate. Most newborns are hypocalcemia or asymptomatic normalization of calcium levels usually occur without intervention within days of birth so no routine screening is not required however symptomatic hypocalcemia can present with jittery respiratory symptoms apnea strider laryngospasms wheezing from bronchospasms and even seizures.

WK3---TUE--->--5yo g present w/abd cramping and pain and diarrhea...it started 3 day ago while at daycare...temp is 100.4,,,abd diffusely tender..stool is positive for occult blood and contains mucus... -most appropriate management?

-close follow up - Bacterial gastroenteritis should be suspected in a patient with bloody or mucoid diarrhea. Treatment is supportive, with close follow up and will appear in children. Empiric antibiotics are not recommended primarily due to the increased risk of hemolytic uremic syndrome associated she could toxin producing E. coli.

WK3---TUE--->--18month old boy presents to ED..drooling..difficuly swallowing...lips and chin red after ingesting oven cleaned..his shirt is covered in oven cleaner.. -NBSIM?

-clothing removal The first step in managing caustic ingestion is assessing airway, breathing and circulation. Contain contaminated clothing should be removed immediately upper Gastro endoscopy is the diagnostic study of choice to evaluate the extent of injury. Attempting to neutralize the alkali wood vinegar or lavage is dangerous as these interventions may trigger vomiting which may cost for the mucosal damage.

WK3---TUE--->-3yo boy fell while playing..has right knee swelling..last week had large bruise on left hip after falling while climbing the living room couch..ecchymosis seen on left hip...abg shows hb of 13...platelets of 187K...XRAy of knee shows large effusion -nbsim?

-coagulation studies -Hemarthrosis after minor trauma is suspicious for a bleeding disorder such as hemophilia and coagulation studies should be obtained. Hemophiliac presents with a prolonged PTT, normal PT and normal platelet count. -Synovial fluid and Gram stain and culture are indicated for septic arthritis which typically presents with refusal to bear weight and size of systemic inflammation such as fever and leukocytosis -Coagulopathy may result from advanced liver disease which impairs production of vitamin K dependent clotting factors 27 910 and often causes thrombocytopenia. -A skeletal survey/full body x-ray is indicated when non-accidental trauma is suspected such as child abuse . Red flags include an inconsistent history of bruises that are pattern such as belt buckles or in unusual location such as abdomen buttocks -Bone marrow evaluation is indicated for spontaneous bruising in patients with anemia and leukopenia or leukocytosis which are finding suggestive of leukemia.

WK3---TUE--->--a newborn that 10minutes old experience respiratory distress...was born at 38wks gestation..PE reveals nasal flaring...grunting..cyanosis..CXR reveals bowels in chest -likely dx?

-congenital diaphragmatic hernia.. Congenital diaphragmatic hernia results in herniation of abdominal contents into the thoracic cavity. Patient have respiratory distress with absent breath sounds on the affected side a barrel-shaped chest in a scaphoid adamant chest x-ray shows intrathoracic bowel loops and displays cardiac silhouette -Group B strep presents with respiratory distress after birth but x-ray shows diffuse alveolar infiltrates and possible pleural effusion's -Tension pneumothorax in a newborn can present with acute onset of respiratory distress, asymmetric breath sounds, and deviation of the heart to the contralateral side -Tetralogy of fallot--- severe right outflow tract obstruction can present shortly after birth cyanosis and a harsh systolic ejection murmur characteristic chest x-ray findings is a boot- shaped heart -Transient tachypnea of the newborn is due to retain lung fluid and present in the first few hours of life with increase work of breathing. Hyperinflation and fluid in the fissures are the classic chest x-ray findings.

WK3---TUE--->-12wk Presents w/ weakness..poor appetite...choking spells during feeding...exclusively breastfed ..PE REVEALs...apathy..weakness...hypotonia...large tongue...sluggish movement...abd bloating...umbilical hernia -likely dx?

-congenital hypothyroidism -Infants with congenital hypothyroidism initially appeared normal at birth, but gradually developed apathy weakness hypotonia large tongue sluggish movement abdominal bloating and umbilical hernia. For this reason screening newborns for hypothyroidism, along with phenylketonuria and galactosemia standard performed in all states. Infant botulism occurs most commonly in diet include honey or canned food. Word Knigge Hoffman syndrome is an autosomal that's a disorder that involves degeneration of the anterior horn cells and cranial nerve motor nuclei. It is a cause of floppy baby syndrome the other cause is botulism. Myotonic congenital myopathy is an autosomal dominant disorder characterized by muscle weakness and atrophy most predominant in the distal muscles of the upper and lower extremities myotonia testicular atrophy in baldness

WK3---WED--->--35 yo nM presents w/ fatigue for past 3wks..increasingly tired...awakens at 4am for no reason...no interest in seeing friend..no energy top play with kids..confused..withdrawn Likely sleep findings?

-dec REM sleep latency -Sleep abnormalities..are core sxs of depression...difficulty falling asleep...interrupted sleep...early morning awakening...and hypersomnia... A characteristic polysomnographic finding an MDD is decreased rem sleep latency meaning the time from sleep onset until the start of the first rem sleep. Which is considered a biomarker for depression. Other sleep findings in depressed patients include decreased slow wave sleep and DEC rem sleep duration and density, as well as disruptions in sleep continuity. Sleep disturbance she's in MDD typically return to normal with antidepressant treatment. -Sleep disturbance is a core symptom of depression. Polysomnographic findings and major depressive disorder include decrease rem latency and decreased slow wave sleep as well as increased rem sleep duration and rem sleep density. Sleep related ventilation and apneas on polysomnography are characteristic of obstructive sleep apnea. These patients commonly have a history of observed breathing pauses, snoring, and gasping for air, as well as excessive daytime sleep rather than early morning awakening.

WK3---WED--->--26yo female was prescribed an antidepressant and says she feels great 2 days after starting the med...little need for sleep...pressured speech thats loud and rapid... -most appropriate initial mgt?

-discontionue fluoextine - The patient's elevated mood, decreased need for sleep, pressured speech, and increase activity energy following exposure to the SSRI, fluoxetine are consistent with antidepressant induced hypomania/mania. All antidepressants carry the risk of inducing mania and patients many patients who experiences are ultimately diagnosed with a bipolar spectrum disorder. In addition to her symptoms suggestive of hypomania/mania, this patient a typically rapid improvement in depression after two days of antidepressant treatment which typically which typically takes about two weeks to work is an additional clue that she may be in the bipolar spectrum. The most appropriate initial step management of anti-depressant induced hypomania is to discontinue the antidepressant to prevent further worsening of the mania. All antidepressants are associated with the risk of inducing mania and patients. The first step in management is to discontinue the offending medication.

WK3---MON--->--18month old baby undergoes cardiac surgery...4days post op develops tachypnea w/ an inc in oxygen requirement....CXR reveal significant pleural fluid via thoracocentesis..pleural fluid show triglycerides of 724 -mechanism causing pt's pleural effusion?

-disruption of thoracic duct.. Chylothorax is an exudative if using due to disruption of the Fattic flow within the thoracic duct. Plural fluid analysis demonstrates milky white fluid with elevated triglycerides. Empyema's are exudative effusion resulting from bacterial infection of the plural space. Plural fluid analysis and impediment show low glucose, low pH, neutral feel predominance with elevated cell count, and positive Gram stain and bacterial culture. Hemothorax occurs due to bleeding into the plural cavity after trauma. DiRocco centesis reveals frankly bloody plural fluid with the hematocrit greater than 50% of the plural fluid. Transudative a fusion result from oncotic or hydrostatic pressure differentials. Protein loss such as nephrotic syndrome, for example, results in decreased plasma oncotic pressure's while increased pulmonary vascular pressures such as cardiac failure, pulmonary embolism cause increase hydrostatic forces both resulting in pleural effusion's. Hepatic hydrothorax can also lead to transition date of infusions when acidic fluid flows from below the diaphragm higher pressure into the pleural cavity lower pressure.

WK3---MON--->-baby w/ thrush..fever..poor weigh gain..diarrhea..signs of resp distress...aspirates reveal PJP poneumopnoia...baby was adopted and mom never went to dr appoint -liklely casue of baby's complicatons

-dt dec cd4 lymphocyte count HIV infection in infancy present with failure to thrive, generalized lymphadenopathy, and opportunistic infection such as numerous as pneumonia, severe thrush. Selective loss of CD4 positive cells is consistent with HIV, and PCR testing confirmed the diagnosis.

WK3---TUE--->-6day old baby born in religious community that doesnt get vaccinations...mom didnt get prenatal care...umbilical cord was cut with a pair of kitchen scissors..temp=101.8....swollen umbilical stump..hypertonicity..neck stiffness..and inspiratory stridor -likely dx?

-dx is tetanus Neonatal tetanus can occur in infants born to an immunized mothers frequently following umbilical stump infection. Affect affected infant present in the first few weeks of life with trismus, spasms, and hypertonicity. Treatment includes antibiotics and tetanus immune globulin. Febrile seizures are usually short generalized seizures status car during periods of fever and young children at age 3 months to five years. omphalitis is a potentially severe cutaneous infection that begins at the umbilical stump with erythema, swelling, and pus and can lead to sepsis.

WK3---WED--->--81 yo F present to f/u for DM...medical dx shows orbital fracture..uncontrolled htn..bruising on forearm..uncontrolled glucose of 220..w/ hbA1c of 9%..she says she cant afford her medication -explanation of clinical findings?

-elder abuse The patient has findings that raise concern for elder abuse including multiple injuries and acute change in financial status such as discontinuing long-standing medication is due to cost, possibly due to diversion of finances by family members. Abuse is most common in older women, especially those who are socially isolated or have a shared living environment living with multiple nonspousal others. Elder abuse is also more likely to explain the patient injury pattern orbital fracture multiple bruises and unusual locations. Manifestations of physical abuse in the elderly may include unexplained pain, injuries in multiple stages of healing, or injuries and unusual locations. Elderly individuals are also at risk for financial exploitation, psychological abuse, sexual abuse, and neglect. Nutritional deficiencies are usually seen in elders who live alone and do not have outside nutritional support. A variety of vitamin C deficiency can cause bruising.

WK3---WED--->--32 yo M presents w/ sxs of schizophrenia.. -what is expected to be seen on brain imaging?

-enlargement of cerebral ventricles Enlargement of the lateral cerebral ventricles is the most consistent replicated Nuro imaging findings and schizophrenia. Nuro imaging studies have frequently shown loss of cortical tissue volume with ventricular enlargement in a subset of patients with schizophrenia, with lateral ventricular enlargement being the most widely replicated binding. Atrophy of the caudate huntington disease a progressive neurodegenerative disorders characterized by chorea, dementia and psychiatric symptoms Decrease volume of the hippocampus and amygdala has also been associated with schizophrenia Accelerated head growth during infancy and increase total brain volume have been found in autism Structural abnormalities in the orbital frontal cortex and basal ganglia are associated with obsessive compulsion disorder.

WK3---WED--->--28 yo M present w/ anxiety after being promoted and having to move to a different office on a higher floor..he;s been fearful of heights since childhood..after learning how his cousin had a sever injury from a fall -appropriat tx?

-exposure therapy This patient's fear of height is consistent with a diagnosis of specific phobia, in which a few specific objects or situations phobic stimulus. The first line treatment of specific phobia is CBT, which involves exposure to the phobic stimulus in a controlled setting. Exposure therapy has been shown to be superior to pharmacologic treatment in specific phobia. Specific phobia is characterized by market fear of specific object or situation. Cognitive behavior therapy using exposure techniques is the preferred treatment. Dialectical behavior therapy is useful for patients who struggle with emotion regulation and distress tolerance and is commonly used to treat borderline personality disorder. Mirtazapine is an atypical antidepressant used in the treatment of major depressive disorder. The selective serotonin reuptake in Hibbett her inhibitor is a first-line treatment for panic disorder and generalized anxiety disorder Trauma focused psychotherapy is a form of CBT used to treat post-traumatic stress disorder. Which presents as intrusive symptoms such as flashbacks nightmares pervasive hyperarousal.

WK3---MON--->--6yo boy presents w/ fever and lump pn neck...rash on trunk and red eyes...temp 103..bp=90/60..red eyes b/L...tachycardia..lips and tongue redness -other findings seen?

-extremity edema Kawasaki disease presents with fever lasting in five days or longer, in addition to four or five mucocutaneous findings such as conjunctivitis, mucositis, rash, distal extremity changes such as extremity edema and cervical lymphadenopathy. Best initial tx=high dose aspirin and IVIG -Always consider Kawasaki disease in small children with a rash and high fever unresponsive to antibiotics. "CRASH and BURN": Conjunctivitis, Rash, Adenopathy(swollen glands/lymph nodes), Strawberry tongue, Hands and feet, and BURN (fever ≥ 5 days) are the most common features of Kawasaki disease. -Sandpaper like skin is associated with scarlet fever which also presents with fever exudative pharyngitis and a sandpaperlike rash most prominent along skin folds such as axilla are groin--scarlet has normal lips and no conjunctivitis -Petechiae would be expected in Rocky Mountain spotted fever which presents with the rash and fever, patients often have abdominal pain and the rash typically spread from the extremities towards the trunk.

WK3---MON--->--5hr old boy presents w/ minimal arm movement...dt difficulty removing shoulder during delivery...baby weigh iOS 9lb and 14oz...mom has missed most of prenatal appopintments -RFs that predisposed baby to complications?

-fetal hyperglycemia w/ evince of macrosomia baby..weighing 8lbs and 13oz or more Excessive maternal hyperglycemia causes fetal hyperglycemia and hyper insulinemia. Neonate may be macrosomic, increasing the risk of shoulder dystocia and its complications such as brachial nerve palsy and clavicle fracture. Fetal growth restriction in small for gestational age are infants with a small birthweight less than 10 percentile.

WK3---WED--->--21yo F presents w/ inc irritation of having breast..considering wearing a chest binder to make breasts flatter...dont want to be seen as a woman because never felt like a woman..moved away from family dt feeling too confined -what explains pt's condition?

-gender dysphoria w/ e/o not identifying w/ assigned gender..distressed dt growing secondary sexual features...strong persisted desire to live according to their experienced gender.... Gender dysphoria is characterized by persistent distress due to incongruence between gender identity and assigned gender. It is often accompanied by discomfort with development of unwanted sexual secondary characteristics. Gender identity affirming care can include psychosocial support, hormonal therapy, and gender affirming surgeries. Gender non-conformity is when individuals do not conform to stereo typical gender roles but still identify with their assigned gender Adjustment disorder is diagnosed when symptoms occur within three months of an identifiable stressor. Patients were depersonalization disorder experience recurrent feelings of detachment or estrangement from oneself such as a sense of being an automation or an outside observer, typically in response to acute or chronic trauma. Disassociative identity disorder is a severe disassociative disorder characterized by switching amongst two or more distinct personalities states and an extensive inability to recall important personal information.

WK3---MON--->--8yo boy presents to the ED dt bee sting..right hand is swollen w/ redness and induration extending 11cm from a central clear area -cause of its sxs?

-large local reaction Patient with stains from homOtero species can develop exaggerative local allergic responses called large local reactions, characterized by swelling erythema and warmed up to 10 cm in size contiguous with the side of the beast thing. Compartment syndrome presents with pain out of proportion, paresthesia, In pain with passive stretch. Lymphangitis occurs when bacteria from a distal side seed then in lymphatic channels, causing erythematous, painful streaks in the skin and fever. Systemic anaphylaxis is a sudden life-threatening I GE mediate response to certain substances. Also systemic symptoms seen in anaphylaxis such as vomiting, Airway swelling, respiratory difficulties, shock Toxic reactions occur in patients usually infants or elderly who have sustained evaluation from numerous things by members of the home interior species. Symptoms such as nausea vomiting diarrhea seizures fevers are due to direct basil at the properties of the venom.

WK3---MON--->--4yo girl presents wq/ worsening cough and nasal discharge...2wks ago had nasal congestion and runny nose..which initially improved over a few days...for past 10 days had thick, yellow-green nasal discharge...worsening daytime cough and waking up at night w/ cough temp is 99 -NBSIM?? -Likely dx? -Supportive evidence along with other common features? -Tx?

-give oral abx Acute bacterial rhinosinusitis is distinguished from viral upper respiratory infection by the presence of severe, persistent, or worsening symptoms such as cough nasal discharge. Treatment is with oral antibiotics. The patient has worsening daytime cough..nasal discharge consistent with acute bacterial rhinosinusitis, A common complication of viral upper respiratory infection. Viral URI symptoms typically last 7 to 10 days. In contrast, acute bacterial rhinosinusitis is diagnosed by any one of the following three criteria: Number one: symptoms such as cough or congestion persist for 10 days or more without improvement Or symptoms severe..such as onset fever 39°C or higher plus drainage for three or more days Or symptoms worsen following initial improvement such as biphasic illness. Fever may be absent Acute bacterial rhinosinusitis Sinusitis → Clinical features Acute bacterial sinusitis An inflammation of paranasal sinuses and adjacent nasal cavities caused by a bacterial infection; most commonly S. pneumoniae, H. influenzae, or M. catarrhalis. Symptoms include purulent nasal drainage and nasal obstruction or facial tenderness for up to 4 weeks. Clinically defined by a duration of > 10 days or worsening after initial improvement (double worsening).

WK3---TUE--->--14yo M present w/ sever L ear opain..past 3 yrs had recurrent acute otitis media complicated by sinusitis...labs showed IgM of 40(normal is 38-266)IgG=258(normal is 768-1728) IgA=28(normal:99-396). Received vaccinations but titers remained low -following defect likely present?

-impaired differentiation of mature B cells -Common variable immunodeficiency is characterized by decreased production due to impaired differentiation of B cells into plasma cells. Patients typically develop recurrent Sino pulmonary infections after puberty and have low IgG plus low IGA and or IGM as well as decreased antibody response to vaccines. -X-linked agammaglobulinemia is characterized by absent b cell production due to a defect in BTK gene. All immunoglobule in levels and vaccine tithers are marked low or absent..lymphoid hypoplasia tissues.. Presentation is typically an infant boys after maternally derived IgG wears off around 6 months old and there is often a family history of recurrent hospitalizations. -Classic presentation of terminal complement deficiency is recurrent neisseria infection -CGD characterized by impaired neutrophil chemo taxis, leading to frequent skin and respiration infections.

WK3---MON--->-4month old boy prconditionss to ED w/ seizures...3 episodes of upper respiratory ifxn since birth...lethargy..weight less than 5th percentile...round cheeks like a doll face..abd distention/hepatomegaly..elevated lactic acid..hypogylcemia -likely cause of condition?

-impaired glycogen to glucose conversion Glucose six phosphate taste efficiency also known as type one glycogen storage disease Van Gogh disease presents at 3 to 4 month old with hypoglycemia often with seizures, lactic acidosis, hyperuricemia, and hyperlipidemia. Patients typically have a doll light face/puffy cheeks, then extremities, short stature, and protuberant abdomen due to Pato Meghani hepato-megaly The inability to break down bury acid chains is a feature of battery acid cycle disorders such as medium chain a circle a dehydrogenase deficiency MCAD. M cad results in episodes of hypo ketotic hypoglycemia during fasting states such as illness.

WK3---TUE--->-3month only baby..exclusiovely breast fed..presents w/ constipation..hypotonia...BL ptosis, sluggish pupillary run to light...copious drool..weak suck and gag reflexes and head lag..extremities flaccid on extension...mom says they been enjoying stroller walks in their newly built California neighborhood -mechanism of baby's illness?

-ingestion of clostridium botulinum spores The diagnosis should be expected in infants with bilateral bulbar palsy's such as ptosis sluggish pupillary response to light, poor sucking and gag reflex is followed by symmetric descending flaccid paralysis/hypo Tonea. Constipation and drooling due to autonomic dysfunction also occur. Infant botulism is caused by the ingestion of clostridium botulinum Ness scores which colonize the guide and lead to production and release of a neurotoxin responsible for life-threatening, descending flaccid paralysis. The diagnosis should be considered when a previously healthy and being present with bulbar palsy's, constipation, and hypo Tonea, even if the infant has not been fed honey. Human derived bachelors in immune globulin is the treatment of choice. This is also common in California Utah and Pennsylvania. Spinal much spinal muscular muscular atrophy were Knigge Hoffman disease is characterized by generalized symmetric proximal muscle weakness and hypo reflexa the weakness is greater in the lower than upper extremities. Dt SMN1 genie mutation..cause lower motor neuron sxs..casue anterior horn motor nuclei degeneration

WK3---MON--->--6day oldgirl presents w/ jaundioce..that began on her face and has spreaded to her abdomen...and to the palms..skin show generalized jaundice...lab results at 24hrs of life showed hemoglobin of 15.7 and total bilirubin of 13..mchc 42% reticulocytes elevated at 6%(normal is 1.8-4.6)..red cell distribution is elevated at 18%9 (normal 11.5-14.5%)

-likely cause of baby's anemia? Defect in the red blood cell membrane Hereditary spherocytosis caused by a defect a red blood cell membrane proteins, often present with refractory jaundice in splenomegaly on examination and laboratory evidence of hemolytic anemia with an increase MCHC. Pyruvate kinase deficiency and glucose six phosphate dehydrogenase deficiency can cause hemolytic anemia due to inherited defects of erythrocyte glycolytic enzymes.

WK3---TUE--->-11yo Bioy presents w/ scoliosis..long finger,,,upward eye lens..crowded teeth...joint hyper mobility..skin hyperplasticuty...diastolic murmur -etiology?

-mutation în fibrilin-1-gene Marfan syndrome is an autosomal dominant disorder that results from mutation of the fibrillation one gene. It affects patients affected patients have tall stature long thin extremities long endurance joint hypermobility upward lens dislocation and aortic root dilation. Ehlers Danlos syndrome is a collagen disorder characterized by scoliosis joint laxity and aortic dilation Congenital contractural arachnodactyly is an autosomal dominant condition resulting from mutations of the fibrillin-2-gene. These patients have tall stature long fingers multiple contractures involving large joints ocular and cardiovascular symptoms are not present. Non-disjunction resulting in an extra X chromosome describes Klinefelter syndrome which presents as tall and slender stature.

WK3---TUE--->--15yo girl presents w/ primary ammenorrhea...w/ no secondary sexual charachteristics(breast tanner1)..height in the 3rd percentile... -NBSIM?

-need a pelvic ultrasound Because ammenorrhea can result from either functional or anatomical problems of the hypothalamus, pituitary gland's, ovaries, uterus, or vagina. Therefore, the first step in management is to evaluate the female reproductive tract with a pelvic ultrasound to exclude anatomic abnormalities of the ovaries, uterus, or vagina. Primary amenorrhea is the absence of menstrual cycle at age 15 or older in girls with secondary sexual characteristics such as breast development or age 13 or older in girls without sexual secondary characteristics. The first step in management is a pelvic ultrasound to evaluate the female reproductive tract. -Testing of 17 hydroxyprogesterone in TSH evaluate for primary amenorrhea secondary to congenital adrenal hyperplasia and hypothyroidism -Brain MRI is performed in patients with a uterus and a low or inappropriately low to normal FSH such as to evaluate for pituitary or hypothalamic tumors as a cause of amenorrhea -A pelvic ultrasound reveals a pre-pubertal uterus with small ovaries FSH level is 60 normal is less than 40, LH is 30

WK3---TUE--->-16yo M had head injury..was dazed and disoriented w/ mild HA...but never loss conscious and appears tired,,, -likely cause of sxs?

-neuronal functional disturbance This patient who developed disorientation after a head injury but does not have sustained neurologic deficit likely has a concussion, a form of mild Trumatic brain injury. Head trauma typically due to blood force can cause widespread neuron of the polarization, decreased cerebral blood flow, and localized lactic acidosis which leads to transient disturbance of normal neuron of function. Concussion is a form of mild Trumatic brain injury resulting in transient impairment of normal neuron or function. Typical symptoms include headache, disorientation, dizziness, and or amnesia associated with abnormalities in coordination, speech, attention, or emotions. Diffuse axonal injury's occur with tearing of the white matter tracks. It typically occurs with severe trauma such as shaken baby syndrome had impact during car collision, and patients are severely obtunded or comatose. Beseler skull fracture typically present with battle sign such as bruising behind the ears or raccoon eyes such as. Orbitals bruising and may be complicated by cerebral spinal fluid leak which can cause clear rhinorrhea Epidural hematoma is commonly caused by rupture of the middle meningeal artery due to fracture of the temporal bone. A temporal hematoma is often present, and patient frequently experiences a transient loss of consciousness followed by a lucid interval before declining Tacoma. Subdural hematoma is caused by tearing of the bridging veins. Traumatic subdural hematoma usually presents with, at the time of injury; those who remain lucid typically develop vomiting, cranial nerve palsy's, and ataxia before declining to coma.

WK3---MON--->-2yo boy presents w/ parents after losing consciousness earlier today after falling oil outstretched hands while playing..he didnt hit his head...the pt gasped and became pale and limp...and diaphoretic and loss conscious..bp is 90/50... whats needed to establish dx?

-no testing needed bc pt had a breath holding spell.. Pallet breath holding spells are triggered by fear or pain from minor trauma and are characterized by pallor/diaphoresis with brief loss of consciousness that may be followed by sleepiness/confusion. Diagnosis is clinical and requires no testing. Ambulatory ECG a vent monitoring is indicated for patient with a family history of cardiac disease or sudden death should go should undergo an ECG to evaluate for arrhythmias such as prolonged QT syndrome A CT scan of the head is perform for suspected brain injury in patients with altered mental status and loss of consciousness after head trauma. Echocardiogram can assess for tetralogy of fallot which presents with a harsh systolic murmur from pulmonary stenosis

WK3---TUE--->-15month old baby cry...turns blue and become unresponsive and parents take away toys because its bed time -dx assoc w?

-normal development..its a breath hold spell Breath holding spells are characterized by brief apnea and color change such as cyanosis or pallor associated with emotional trigger and followed by loss of consciousness. Parent should be reassured that these pills are typically benign with no long-term consequences. -ECGs are used to evaluate for arrhythmias such as prolong QT syndrome -Orthostatic hypotension due to volume depletion is a common causes of syncope with postural changes in older children and adults. -Tetralogy of fallot at presents with cyanotic episode such as Tet spells after crying along with the rapid and deep breathing occurs ...a harsh systolic ejection murmur due to right ventricular outflow tract obstruction would be expected

WK3---MON--->-4yo M presents w/ rash/pinpoint bruises all over his body for past 24hrs..3wks ago had a viral illness that resolved..oral mucosa is normal...hb=13.5...platelets=40K...blood smear shows few platelets that varied in size and morphology -NBSIM?

-observation only!! Kid has ITP(idiopathic thrombocytopenia purpura)..pt's platelet count is over 30K and there is no bleedind..so no need to do anything... but if platelet count is under 30K w/ sever bleeding..then next step would be corticosteroids or IVIG Immune thrombocytopenia typically presents with isolated thrombocytopenia after a viral infection. Children with isolated cutaneous symptoms such as PTA usually recover spontaneously and require observation alone, regardless of platelet count. Those with bleeding should receive steroids, anti-D, or intravenous immunoglobulin. Bone marrow evaluation is performed in the presentation of concerns for leukemia such as fever, leukopenia/leukocytosis, anemia.

WK3---TUE--->-16yo boy present w/ left leg pain...it interrupts his sleep but improves w/ ibuprofen. -likely dx?

-osteoid osteoma - ostroid osteoma is a benign bone forming tumor that presents with increasing pain that is worse at night and is unrelated to physical activity; the pain improves with NSAID medication. Radiographs demonstrate a single small round lucency. Ewing sarcoma typically presents in the pelvis or long bone diaphysis example femur with pain and swelling. Pain is generally worse with activity and does not respond to NSAIDs x-ray shows extensive bony involvement with onion skinning due to new subperiosteal bone formation Osteomyelitis typically presents with pain, Decreased mobility, and swelling in addition to fever x-ray may show soft tissue swelling/Adema or periosteal elevation Our osteosarcoma typically presents with chronic localized pain and soft tissue mass x-ray show Bonnie destruction with a Sunburst pattern of periosteal reaction such a spicules of ossified fibers

WK3---WED--->--25yo M had a psychotic episode and destroyed a tv in the store..was taken to hospital and received a haldol injection...hours later he has sustain contraction in his neck... -nbsim?

-prescribe diphenhydramine - The patient experienced acute dystonia which is characterized by muscle spasms or stiffness in the head and neck it also include tongue protrusion or twisting, facial grimacing, torticollis, muscle spasms in the back and oculogyric crisis such as Forrest or sustain elevation of the eyes in an upright position. Acute dystonia reactions are treated with anti-cholinergic medication such as benztropine and antihistamines such as diphenhydramine. acute dystonia is a type of EPS associated with antipsychotic treatment. It is most commonly seen in hypotensive first generation antipsychotics and space treated with anti-cholinergics such as BENZTROPINE or anti-histamines such as diphenhydramine. Amantadine is a dopaminergic medication used to treat drug induced parkinsonism. Dantrolene is a muscle relaxer that has been used in severe cases of neuroleptic malignant syndrome and malignant hyperthermia neuroleptic malignant syndrome is a life-threatening condition characterized by high fever muscle rigidity and rhabdomyolysis. Levodopa is a treatment for Parkinson's disease. If side effect of antipsychotic use, the preferred treatment is amantadine or BENZTROPINE AKATHISIA may be treated with beta blocker such as propranolol and BENZODRAZEPAINES such as lorazepam. VALBENAZINE is used to treat tardive dyskinesia.

WK3---MON--->--16yo girl comes to the ER...last menstrual cycle a week ago..concerned about being pregnant... -most appropriate mgt of patient?

-provide levonorgestrel pill The levonorgestrel pill such as plan B Are readily available and effective emergency contraception option that prevent pregnancy by delaying ovulation. In most states, adolescent seeking pregnancy prevention options may receive confidential medical care without parental consent.

WK3---MON--->--a newborn undergoing screening...delivery was complicated by meconium stained amniotic fluid...hr is 120/min and RR=50(tachypnea is over 60).. -what's indicated during its hospital stay?>

-pulse oximetry Routine newborn care includes screening for critical congenital heart disease such as ductal lesions, cyanotic heart disease with pre-and post ductal pulse oximetry in all events in infants Chest x-ray is indicated for suspicion of meconium aspiration which presents with tachypnea respiratory rates over 60 per minute increased work of breathing such as retractions and cyanosis Echocardiogram evaluate for congenital heart disease in a patient with a positive congenital heart disease screen or cyanosis Electrocardiogram is indicated in a patient with bradycardia and suspected neonatal heart block. Hyperoxia test such as administering 100% oxygen can distinguish between cyanosis due to pulmonary disease which improves with oxygen and congenital heart disease where you will see persistent cyanosis.

WK3---MON--->-13 yo boy presents w/ parents about concern of his behavior...recently got in trouble for forging signatures to go on fieldtrip...cigarette lighter found in pocket but says he doesnt smoke...accused of being involved a suspicious fire at a park...always wanted to be a firefighter...he says he sets fires just to see how long it takes for firefighters to arrive -likely dx?

-pyromania Pyromania is characterized by intentional and repeated fire setting with no obvious motive. Patients have a fascination with fire and deliberately start fires to reduce tension and feel pleasure or relief. Oppositional defiant disorder is characterized by a pattern of angry or irritable mood and defiant behavior towards authoritative figures. A diagnosis of conduct disorder requires a pervasive pattern of violating rules and the rights of others that includes additional problem behaviors such as aggression/cruelty towards people and animals and theft.

WK3---TUE--->--5month old boy 2day hx off vomiting...diarrhea...generalized tonic-clonic seizure..exclusively formula fed from birth...fair hair..fair skin..blue eyes...musty body odor -appropriate diagnostic tool?

-quantitative amino acid analysis Phenylketonuria is caused by a deficiency of phenylalanine hydroxylase, which results in the buildup of phenylalanine and its Nuro toxic metabolite. Clinical features include intellectual disability, fair complexion, eczema, and musty mousy body odor. Infants diagnosed early to newborn screening and treated with low phenylalanine diet can expect normal health and develop. -Aldolase B-deficiency causes hereditary fructose intolerance. The introduction of fruits and vegetables into the diet is followed by the accumulation of fructose one phosphate. Affects infants can present with vomiting poor feeding and lethargic. Seizures or encephalopathy follow if fructose is not removed from the diet. -Complete absence of galactose-1-phosphate uridyl-transferace activity in the red blood cells is consistent with galactosemia. Galactosemia are usually present in the first few days after birth with jaundice, hepatomegaly, and failure to thrive after consumption of breast milk or regular milk. - Karyotype can diagnose numerical or structural chromosome abnormalities such as down syndrome and turner syndrome and structural such as fragile X syndrome.

WK3---WED--->--61 yo attorney has been sleeping restlessly...waking up startled and sweating since he suffered from a MI 2months ago in the court room..he also has avoided returning to work.. -nbsim?

-recommend CBt Life-threatening medical events may be experienced as traumatic and result in posttraumatic stress disorder. Trauma focused CBT is the preferred initial treatment.

WK3---THURS--->--22yo F present to ED w/ family bc decloingin grade...getting messages from tv..paranoid at work ..was dx w/ schizophrenia and prescribed respridone -next help measure for pt?

-recommend family therapy - Family psychosocial interventions are indicated for patients with a recent psychotic episode who have experienced ongoing contact with family members. Minimizing conflict and stress in the home decreases the risk of relapse in patients with schizophrenia.

WK3---THURS--->---a hospitalists seems difficult to reach..distracted and irritable during discussions and leave pt notes incomplete for longer then 48hrs..he rounds late at 11pm.. -most appropriate step to take as his colleague ?

-report the dr to the hospital physician health program -physicians are ethically obligated to report suspected physician impairment to protect patient safety. Physician Health Programs are organizations to designated to investigate reported concerns and arrange for comprehensive assessment and treatment id necessary. Physician impairment clues include..incr difficult to reach...late rounding...inc irritability

WK3---THURS--->--32yo M present to ED w/ mom bc he's been staying up late overnight for the past week...shouting to God..believes he has special powers...pmh of MDD w/ suicidal ideation.. Likely dx?

-schizoaffective The diagnosis of schizoaffective disorder requires assessing the longitudinal course of the illness and determining if the patient has had at least two weeks of psychotic symptoms in the absence of a mood episode. Affective disorder is distinguish from schizophrenia by the present a mood symptoms for significant portion of the illness.

WK3---WED--->--22yo M is brought to office by mom bc..he has no friends..does not dat..and shows no interest in activities popular w/ young adults...he works and comes home and enjoy computer games ..during interview he has a flat affect..limited eye contact...and states he rather keep to himself -behavior consistent w/?

-schizoid personality disorder The patient's pattern of lack of desire for close relationships and preference for solitary activities is characteristic of skills white personality disorder. Individuals with this disorder typically have your friends by choice it's a little interest in intimacy or sexual experiences. They tend to be emotionally detached with flat affect and apparent indifference to praise or criticism. Individuals with schizo personality disorder are often considered loaners tend toward converted behaviors. Individuals schizoid personality disorder are socially detached prefer to be alone. They can be differentiated from individuals with avoidant personality disorder, who desire relationships but avoid them due to fear of rejection. They lack the eccentric cognition and perceptual distortions characteristic of schizotypical personality disorder. Individuals avoidant personality disorder or social anxiety disorder desire relationships but avoid them due to fear of rejection or being embarrassed.

WK3---THURS--->---parents bring 15yo boy to the ED dt changed behavior..wathderawn...hallucination...poor eye contact..poor hygiene... -likely dx?

-schizophreniform - Schizophreniform disorder is differentiated from schizophrenia by the duration of symptoms. Schizophreniform disorder, symptoms must last for more than one month but less than six months. The diagnosis of schizophrenia requires symptoms to persist for six months or longer.

WK3---TUE--->--6month old baby presents w/ facial rash that spreading to scalp..eyebrows...nasolabial folds..behind ears..not itchy -likely dx?

-seborrheic dermatitis - Seborrheic dermatitis an infant is characterized by a scaly oily rash affecting the scalp, eyelids, nasal labial folds, post or regular area, and umbilicus. Spontaneous remission is common. First line treatment includes emollient and non-medicated shampoo's. Such as selenium sulfide shampoos -A topic dermatitis is a recurrent rash that can affect the cheeks scalp trunk and extensor surfaces and infants atopic dermatitis is associated with severe pruritus a most patient will have a family history of atopic disorders such as asthma or allergic rhinitis. -Contact dermatitis is an inflammatory skin condition caused by contact with an allergen or irritant. Symptoms are similar to atopic dermatitis such as severe pruritus although they are typically confined to a specific exposed area such as...perioral or hands -Psoriasis is a chronic inflammation inflammatory disorder that affects the extensor surfaces of the elbows and knees it is red sharply demarcated scaling lesions that form round or oval plaques or classic presentations..typically happens in adulthood -tinea capitis is a fungal infection of the scalp that causes itchy patches fine white scales

WK3---TUE--->-2yo F presents w/ fever..lethargy...temp is 103.6...bp 70/30..legs are pupuric..csf reveal gam negative diplococci -additional study obtained?

-serum fibrinogen - Purpera is a suggest.... DIC which is characterized by clotting factors in platelet consumption. Laboratory evidence of DIC include thrombocytopenia.. elevated PT PTT and low fibrinogen. A positive anti-nuclear antibody seen in systemic lupus which can cause fever and arash due to thrombocytopenia by vasculitis Dermatologic presentation of group a strep pharyngitis is associated with increased antistreptolysin O antibody Patient with a terminal complement deficiency C-5 C6 C7 C8C9 are at increased risk for invasive infection with encapsulated organisms such as neisseria meningitidis Skin biopsy may be necessary to confirm the diagnosis of HSP which can present with a purpuric low extremity rash

WK3---SUN--->---88yo F present w/ vaginal bleeding and pelvic pain for past 3 days..refuse to leave her room or shower...BMI is 36...vulva is edematous..tender...and friable posterior perineal laceration..US shows endometrial thickness of 3mm(normal is 4 or less).. -nbsim?

-sexual abuse screening RFs for elder sexual abuse includes...living nursing home...over 80yo...somatic sxsx such as pelvic pain..behavorial changes such as refusing to bathe..signs of genital trauma perianal lacerations....must screen for sexual abuse and immediately report to adult protective services.

WK3---SUN--->---How to differ Acute fatty liver of pregnancy/pre-eclampsia v HELLP syndrome(hemolysis, elevated liver enzymes, low platelets)

-signs of hepatic insufficiency(hypoglycemia,encephalopathy) and abnormalities in coagulation profile point towards a diagnosis of AFLP. -liver bx is gold standard for dx -treatment is maternal stabilization(fluids..glucose...possible transfusion if needed) and prompt delivery

WK3---MON--->-7yo boy presents w/ fever and right leg pain...pt has been limping for 3 days...rates pain as 7/10..no relief from ibuprofen...temp 102.9...PE shows bony tenderness and erythema and warmth over proximal tibia...blood culture done -which organism is responsible for sxs?

-staph aureus w/ e.o osteomyelitis...which is the common organism for ppl w/ no underlying problems/school age children Osteomyelitis is a bacterial infection of the bone they typically present with fever and Focal bony pain. Infection is most often caused by hematogenous spread of staff aureus. E. coli and strep agalactiae is associated with neonatal osteomyelitis Pseudomonas associated with osteomyelitis of the foot after a puncture wound of the plantar surface. IV drug users are also at increased risk of pseudomonas osteomyelitis Because of their impaired splenic function, patients with sickle cell anemia are at increased risk of osteomyelitis caused by encapsulated organisms such as salmonella species. symptoms of sickle cell would have patient with a history of vaso-occlusive crisis Stapg epidermidis is associated with patients who have indwelling catheter's such as hemodialysis Strep pyogenes commonly causes cellulitis in septic arthritis

WK3---SAT--->---33yo F at 28wks GA present for routine visit...pmh of DM...last glucose test was 200...growth US shows baby is the size of a 24wk fetus...head size below the 4th percentile...amniotic fluid index is 3(normal is over 5) -nbsim?

-umbilical artery doppler -baby has signs of fetal growth restriction...which could cause placental insufficiency..dec blood flow o2 and dec nutrients to baby..fetal grow restriction also assoc w/ oligohydraminos(when amniotic fluid is 5 or less)..a doppler US can detect proper blood flow to placenta and also determine if urgent delivery is necessary to prevent fetal demise.. Education obj: Maternal Vascular dz like htn, dm,anemia,uterine infxn. can cause placental insufficiency...presenting w/ FGR and oligohydraminos...dt risk of fetal demise..pts w/ FGR require umbilical arty doppler US to assess placental perfusion and the need for urgent delivery. -cervical nucleus acid amplification testing is done at initial prenatal visit to test for chlamydia and gonorrohea and at 3rd trimester if mom is high risk..under 25yo.. -fibrinectin testing-determines risk of preterm delivery in its w/ preterm contractions.

WK3---WED--->--17yo M presents w/ vomiting...abd pain...watery diarrhea for 24hrs..difficulty sleeping for past 2 days...diaphoretic,thin boy in considerable distress -likely dx?

-withdrawal from heroin - Patient has vomiting diarrhea sleep disturbance myalgia and diaphoresis symptoms consistent with abrupt abrupt opioid withdrawal following prolonged use such as heroin. Patients with opioid dependence typically develop withdrawal symptoms within 4 to 48 hours of the last dose symptoms primarily arise from non-educated hyperactivity in the locus see aer ULEUS, Aponte nucleus involved in wakefulness breathing in autonomic function. When opioids are stopped at the chronic use the loss of their usual inhibitory effect lead to sudden increase in New Albany Efron. This results in withdrawal symptoms which commonly include sleep disturbances nausea vomiting abdominal pain such as cramping diarrhea arthralgia/myalgia on examination patients may be restless and irritable. Vital signs are often normal as in this patient. Symptoms of opioid withdrawal include vomiting abdominal pain diarrhea sleep disturbances myalgia. In contrast to other withdrawals states such as alcohol in benzodiazepines which present with similar symptoms in opioid opinions heart rate blood pressure and temperature often normal change.

WK3---THURS--->---mom brings her 30month old daughter to be seen after teacher reports communication deficits...teacher reports that girl only speaks 1 phrase words..although she's almost 3...prefers to play alone... motor skills are normal and mom reports that other sibling was a late talker. -most appropriate response by physician ?

-you're correct that kids' milestones vary..however..I would like to obtain further evaluation so see if intervention is necessary -early intervention in autism spectrum disorder in the preschool and school aged yrs has been shown to significantly improve outcomes. If there is any concerns about ASD..a thorough screening and evaluation should be undertaken and individualized educational/behavioral Services offered as soon as possible

WK3---FRI--->--the npv of a pt having a dz is 96%...what is the probablity of the pt having a dz?

100-96=4% if a test result is negative,...the probability of having the dz is 1-NPV

WK3---THURS--->--19 yo constantly thinks he running over someone when he driving..he alway get out of his car to look when he passed over bumps in the road...the pt always check to news to see if any reports were made.

1st line pharmacotherapy affect which neurotransmitters? -Serotonin Serotonergic antidepressant such as selective serotonin reuptake inhibitor's are the first line medication treatment for obsessive compulsive disorder.

-molar pregnancy

=characterized by 1st trimester vaginal bleeding assoc w/ expulsion of vesicles and uterine size greater than date. US shows "snowstorm" appearance w/ no fetal hr or identifiable structures

WK3---FRI--->--hazard ratio

A measurement of relative risk used in statistical analyses. Calculated as the ratio of an outcome occurring in an exposed group compared to an unexposed group. A hazard ratio greater than 1 indicates increased risk and a hazard ratio less than 1 indicates decreased risk. If the hazard ratio is 1, the risk of the outcome is the same in the two groups being compared. HR less than 1 means event is more likely to occur in control group HR greater than 1 means event is more likely to occur in treatment group education objective: HRs are proportions that indicate the chance of an event occuring in the treatment group compared to the chance of the event occuring in the control group. When revieweing a drug advertisement,it is important to critically read all the presented info

WK3---TUE--->-9yo girl w/ persistent fever,rhinorrhea and congestion for past 2weeks.. sxs began to improve but now remaining persisten

Acute bacterial rhinosinusitis Sinusitis → Clinical features Acute bacterial sinusitis An inflammation of paranasal sinuses and adjacent nasal cavities caused by a bacterial infection; most commonly S. pneumoniae, H. influenzae, or M. catarrhalis. Symptoms include purulent nasal drainage and nasal obstruction or facial tenderness for up to 4 weeks. Clinically defined by a duration of > 10 days or worsening after initial improvement (double worsening).

WK3---MON--->-7month old boy presents to office dt streaks of blood in his stools—switch to rice cereal and cows milk a month ago from being exclusively breastfed..pooping pellets and anal fissure in 6oclock position. Nbsim?

Add prune puree diet dt anal fissure dt constipation Functional constipation in infants usually presents after introduction of solid food such as low fiber decrease fluids and without alarm signs such as poor growth severe abdominal distention. Initial treatment is with nine digestible osmotically active carbohydrates such as fruit juice/purée.

WK3---WED--->--32yo M brought to Er after pt flew into rage and punched a mirror bc wife was late coming home..wife says he's been working around the clock...she couldn't get a word in..pt has a hx of depression in his 20s -likely dx?

Bipolar disorder The patient's two week history of extreme irritability associated with increased activity, decreased sleep, and positivity resulting in self injury, pressured speech such as wife can't get a word in, and racing thoughts can't shut my mind off suggest a manic episode of bipolar one disorder. He has a history of depressive episode in his 20s is consistent with his diagnosis although the classic presentation of mania includes an elevated or euphoric mood, Many patients and manic episode have irritability instead Patients with manic episode of bipolar one disorder may have an irritable rather than euphoric mood, which may manifest as uncharacteristic angry outbursts and aggressive behavior.

WK3---TUE--->-5day old boy..born at 32 wks has persistent emesis...green and then eventually coffee brown...abd distended and pain on palpation.. -nbsim?

Blood cultures and abx -Necrotizing Intero colitis is a life-threatening condition that causes feeding intolerance abdominal distention G.I. bleeding and pneumatosis intestinalis air in Bowel in premature infants. Initial management is bowel rest in addition to blood cultures and empiric antibiotics. -Necrotizing enterocolitis Abbreviation: NEC A dangerous hemorrhagic inflammation of the intestinal wall that is caused by impaired bowel perfusion and defective immunity. It most often affects premature infants. The pathognomonic finding on abdominal x-ray is pneumatosis intestinalis.

WK3---MON--->-3yo girl had a choking episode during a meal...it persist after neublized albuterla and steroids were prescribed..wheeze noted in right middle lobe -nbsim?

Bronchoscopy - Foreign body aspiration should be considered in a patient with persistent cough or wheezing following a choking episode. X-ray may reveal classic findings of unit unilateral long hyper inflation and mediastinal shift however x-ray findings also may be normal, and bronchoscopy is indicated if clinical suspicion remains high.

WK3---MON--->-13 day old boy presents for wellness check...w/ BL eye swelling and redness and scant amount of watery/mucopurulent discharge.. -most appropriate treatment? -orał erthromycin

Chlamydia conjunctivitis typically presents at age 5 to 14 days with mild eyes swelling chemosis and watery or mucopurulent discharge. Management is oral erythromycin; topical treatments are not affective.

WK3---MON--->-2yo boy present to the ED w/ harsh cough w/ suprasternal retractions.....high pitch noise during inspiration that worsens w/ crying...temp of 100.4 Likely dx?

Croup is a viral and illness most commonly caused by parainfluenza virus. He presents with fever, hoarseness, inspiratory strider, and a harsh, barking, seale-like cough. Tx mild cases w/ cold midst and fluids but moderate to severe cases need nebulized racemic epinephrine -Epiglottitis often found in unvaccinated and may assume tripod position such as leaning forward with hands on knees and neck hyperextended to open the airways. -Bronchiolitis is a lower respiratory tract illness most commonly caused by respiratory syncytial virus. Patients have fever, respiratory distress, and wheezing. -Foreign body aspiration often presents after a choking episode with sudden onset respiratory distress or stridor. -Laryngomalacia, caused by the collapse of supraglottic structures during inspiration, presents with chronic inspiratory strider that begins in the neonatal. It is worse in the supine position.

WK3---THURS--->--language disorder(toddlers..not school age kids)

Diagnostic criteria (according to DSM-V) [3]Persistently reduced ability to learn, understand, and use language (written, spoken, and/or signed), characterized by limited vocabulary for age, incorrect understanding of word meanings, difficulty with sentence structure, and/or incorrect grammar useA significant deviation of language abilities below those expected for age, resulting in impaired communication, social participation, and/or poor academic performance Management: language therapy

WK3---SAT--->---15yo girl presents w/ intermittent lower abd pain..woke up w/ sharp pain radiating to lower back...nausea..vom..not sex active..hgc neg..pelvic US shows 5cm left ovarian mass...appear to be in acute distress w/ sever tenderness upon palpation of abd -nbsim?

Diagnostic laparoscopic -pt has e/o partial ovarian torsion..nasu...vom...intermittent pain that resolves..meaning no sxs bt episodes...mass spotted on US..unilatral pelvic pain...mgt w/ diagnostic laparoscopy for manual de-torsion of the adnexa and removal of any cysts or masses... EO:partial ovarian torsion presents w/ intermittent..self resolving episodes of unilateral pain dt temporary ovarian vessel occlusion. Pt w/ intermittent self resolving episodes of unilateral pelvic pain dt temporary ovarian vessels occlusion.Pts w/ intermittent partial torsion can suddenly develop complete torsion, which requires diagnostic laparoscopy CT of abd and pelvis would be inappropriate bc this is an emergent situation and this would delay evaluation and treatment

WK3---THURS--->--48yo F agreed to get a checkup...she says she hasn't had one in 10 yrs bc the dr always try to find something wrong so they can bill her...she's never been in a relationshiop bf bc she think someone will exploit her for her inheritance...as the physician examines her..she ask why does he need to check everywhere

Dx? paranoid personality diorder...w/ eo fear someone is always trying to take advantage of her..out to get her...and reluctant to confide in anyone Paranoid personality disorder is characterized by a long-standing pattern of suspicion and miss trust of others intentions. The physician patient relationship, it may manifest as difficulty establishing rapport and creating a therapeutic alliance. A delusional disorder the persecution type believe they are being conspired against with symptoms lasting one month or longer. They believe in specific and he'll with delusional intensity such as I'm fixed and unshakable.

WK3---TUE--->-16yo boy presents to ED dt eye injury..was hit in eye w/ paint ball...PER shows proptosis(eye bulging out of socket)..periorbitla edema...and ecchymosis...eyelid reveal rock-hard eye -nbsim?

Emergency surgical decompression Orbital compartment syndrome causes vision loss due to rapidly increased intraocular pressure from trauma. Examination shows a tight orbit sample. Orbital swelling, hard and leads to pupillary defect. Management is immediate surgical decompression to prevent permanent vision loss. -CT videography can be used to diagnose cavernous sinus thrombosis which most commonly occurs secondary to infection such as sinusitis... orbital cellulitis. Symptoms include headache and findings such as. Orbital swelling, proptosis, vision loss, cranial nerve palsy.

WK3---TUE--->-Infant..a few wks old +well appearing+bloody/mucus loose stools= IT IS NOT COMMON TO SEEW KIDS UNDER 6 MONTHS W/ MECKELS!!!!!

Food protein intolerance proctocolitsis: a range of symptoms caused by an immune reaction to various food proteinsCommon antigens: cow milk proteins (lactalbumin or casein, among others), soy protein, egg proteinReactions are either IgE‑mediated (e.g., cow's milk allergy) or non‑IgE‑mediatedClinical findingsAbdominal pain, nausea, vomiting, and diarrheaFood protein-induced proctocolitis (e.g., caused by milk/soy protein)affects primarily young infants (typically manifests at 2-8 weeks of age)clinical diagnosis resolution of symptoms once causative antigen is removed Abdominal x-ray is the next step in management courses for suspected intestinal obstruction such as mala rotation with medical bother us or neck or ties into a kaleidoscope both of which can cause a hematochezia due to progressive intestinal a skinnier air contrast enema is diagnostic and therapeutic for intussception which classically present at age 6-36 months with intermittent colicky abdominal pain or fussiness in an infant and vomiting in addition to current jelly stools. Meckel diverticulum a remnant of vittelin duct containing ectopic gastric tissue can cause severe hydrochloric acid and lead to intestinal ulceration and bleeding. Although painless, bloody stools are classic in older children symptoms under six months old or exceedingly rare and usually are related to associated complications such as intestinal obstruction causing ill appearance vomiting and abdominal distention.

WK3---MON--->--121 yo boy presents w/ HA, nausea for 2days....3wks ago had a skin ifxn for 5days...bp 170/98..periorbital edema..BL pitting edema in Lower extremities...BUN=36...creatinie=-2.2....RBC 20-30/hpf -likely explanation?

Immune complex deposition dt PSGN... A cute post strep glomerulonephritis, a complication of group a strep infection such as impetigo, occurs due to certain epithelial deposition of streptococcal antigen/antibody complexes an accumulation of complement component C3 within these deposits. The subsequent glomerular damage causes hematuria, edema, acute kidney injury and hypertension. -Renal interstitial inflammation occurs in acute interstitial nephritis which can be triggered by antibiotics. Patient typically have fever rash eosinophilia and white blood cells or white blood cell casts on your analysis. Reno tubular extraction can occur with acute tubular necrosis due to say severe pre-renal disease such as volume depletion or sepsis, or nephrotoxic agents such as acyclovir or aminoglycosides. Acute tubular necrosis presents with Marie Brown cast and free Reno to Baylor epithelial cells -Renal vein thrombosis can present with hematuria in proteinuria; however, renal vein thrombosis is rare in children raise factors include nephrotic syndrome, central venous catheter's, and malignancy -Hemolytic uremic syndrome causes a thrombotic microangiopathy that can result in a acute kidney injury such as elevated creatinine and children patients typically have anemia and thrombocytopenia.

WK3---WED--->--17yo M presents w/ sxs of Mdd since breakup w/ girlfriend...unable to do school work..no interest in playing sports he enjoyed..refuse to eat..stalks ex gf social media page..thoughts of hanging himself -nbsim?

Inform parent about the patient suicidal thoughts and hospitalize him immediately with or without parental consent. A teenager with active suicidal ideation must be hospitalized for safety and parents must be informed. Parental consent is ideal but not required for hospitalization.

WK3---WED--->--36 yo F present w/ concerns that she may have a manic episode bc she cant adhere to medication schedule dt working over night and forget doses some days -helpful technique to improve adherence?

Involved patient in selecting medication formulation and dosing frequency from options expire graph management of chronic illnesses they require maintenance phase pharmacotherapy is often complicated by pour in Hirons. When feasible allow the patient to choose the drug formulation, frequency of doses, and timing of administration can improve adherence.

WK3---WED--->--68 yo M presents for f/u after a stroke resulting in right sided weakness and facial droop...he refuses rehab services...barely talks..asks friends not to visit..low energy..difficulty sleeping..says of course Im depressed..anyone in my situation would be but no sucidal ideation -best response?

It's understandable that you are depressed, but there are treatments that can help. Post stroke depression is common in underdiagnosed, and untreated depression is associated with reduce quality of life, decreased adherence to treatment, poor functional outcomes, and increased mortality in patient recovering from stroke. Research supports the early treatment of post stroke depression with antidepressants and or psychotherapy, which resulted in improved physical and cognitive recovery. Post-stroke depression is common, undiagnosed, and associated with increased morbidity and mortality. Patient came back benefit from early treatment with SSRIs.

WK3---TUE--->--laryngomalacia v chaonal atresia

Laryngomalacia Croup → Differential diagnoses A collapse of supraglottic structures during inspiration due to anatomical and/or functional abnormalities. Manifestations include low-pitched inspiratory stridor, swallowing dysfunction, reflux, and, in severe cases, failure to thrive. Laryngomalacia is the most common cause of strider in infants and is due to collapse of the supraglottic tissues during inspiration. Strider increases with exertion such as crying and feeding and improved from positioning. Choanal atresia A congenital condition characterized by a bony and/or membranous obstruction of the posterior nasal passage. Unilateral choanal atresia often presents late with chronic inflammation (e.g., rhinorrhea, congestion) of the affected nasal passage. Bilateral choanal atresia manifests as obstructed nasal breathing with intermittent cyanosis immediately after birth; breathing improves when crying, as it allows the infant to breathe through his or her mouth.

WK3---SUN--->---11yo girls presents w/ vulvr itching...she's not sexually active and has NOT began menstruating...trouble sleeping dt constant itching...PE show thin white skin w/ excoriations extending to the perianal area and small anal fissures -likely dx?

Lichen sclerosis -w/ eo thin white vulvar lesions ...anal fissures EO: LS commonly presents w/vulvar itching in prepubertal girls w/ pruritus...and thin white lesions of the vulva and perianal region...tx w/ topical steroids -it can affect any age..however if post menopausal there's an inc risk of cancer

WK3---SUN--->---27yo F presents for 1st prenatal visit...US show 14wks pregnant w/ 2 intrauterine fetal poles../.cervical length is 4.5cm(short is under 2.5)... -nbsim?

Low dose aspirin -we're giving aspiring bc mom is at risk for preeclampsia dt multiple gestations/carrying twins..which inc her chances by 3folds..othr risk factors include ckd..prior preeclampsia...DM...aspirin is a way to help w/ prevention

WK3---SUN--->---33yo f g2p2 present w/ complaint of BL nipple discharge..been amenorrheic for past 2yrs dt IUD...PE reveals large pendulous breast w/out palpable masses or nipple abnormalities...brown-grey discharge expressed from both..preg test is neg... -nbsim?

Measure of prolactin and TSH levels Physiologic/benign galactorrhea is usually BL...guar neg(no blood)..appears milky..clear..brown..yellow..gray..or green..these ss are NOT concerning for cancer such as no palpable masses..no lymphadenopathy..no nipple or skin changes... most common cause of galactorrhea is hyperprolactinemia dt pituitary..certain meds...Hypothyroidism......therefore evaluation of serum prolactin and TSH levels is required in BL galacortthea

WK3---WED--->--58 yo M presents to ER w/ unusual behavior after given medication 5 days ago for an asthma exacerbation...he states he's afraid to go in his restroom bc he sees evil children in there... -likely dx?

Medication-induced psychotic disorder - The patient exhibit the acute onset of psychosis suggest hallucinations that is temporarily associated with the introduction of a new medication as likely a high-dose glucocorticoid for his asthma exacerbation. His psychosis does not appear to be associated with delirium because he is alert and oriented and oriented or any other psychiatric or medical illness. High-dose glucocorticoids given for allergic, inflammation inflammation, or autoimmune conditions may cause glucocorticoid-induced psychosis, an example of medication induced psychotic disorder. Psychotic symptoms typically occur during the first week of glucocorticoid treatment they can occur at any time. Glucocorticoids can also cause manic or depressive symptoms. The core feature of delirium is fluctuating cognitive impairment, characterized by poor attention and disorientation.

WK3---MON--->--8yo boy present w/ difficulty at school..annoying behavior..trouble sitting and focusing...easily irritable...fidgety and interrupts conversations... -most effective med?

Methylphenidate—to tx adhd for school aged kids.. First line treatment for aDHD in school age children is which stimulant medicine is it stimulant medication such as methylphenidate and amphetamines due to their efficacy safety and tolerability Atomoxitine is prescribed when parents want non stimulants for kids

WK3---SAT--->----37yo F who's 8wks pregnant came to the clinic a few days ago and hCg level was 27k...she returned today and hcg level is 25k..PE no vaginal discharge..no bleeding..no cervical tenderness..U/S showed gestational sac w/ yolk sac but no fetal pole(no embryo) -likely dx?

Missed abortion -missed abortion is a prep loss of less than 20wks gestation prior to expulsion of POC(products of conception). Pt w/ missed abortions are typically asymptomatic but can present w/ loss of preG sxs(dec nausea/vomiting) or light bleeding. Findings include a closed cervix, dec BHCG levels...and a US revealing a nonviable (no heartbeat)intrauterine pregnancy -incompete abortion presents w/ pain..bleeding..dilated cervix...and passage of some POC

WK3---MON--->-10 yo boy w/ 10 day hx of thick and persistent nasal discharge, congestion and cough -like cause of condition?

NOntypeable H.Influenza Nontypeable H influenza strep pneumonia and morexalla are the most common bacterial causes of acute bacterial rhinosinusitis, which commonly presents with 10 days or more persistent nasal discharge and cough.

WK3---MON--->-16yo boy w/ Crohn dz presents w/ watery diarrhea..pruiritc rash thats dry and hyper pigmented w/ demarcated lines for last 3 months..also note difficulty concentrating in school -cause of vitamin def

Niacin - Niacin vitamin B3 deficiency is characterized by diarrhea, photo-sensitive dermatitis, and neurologic symptoms ranging from impaired concentration to dementia. Severe cases can be fatal. Deficiency of cobalamin B12 causes macrocytic anemia glossitis in neuropsychiatric symptoms such as shuffling gait. Seizures depression. Pyridoxine vitamin B6 deficiency is rare and can cause irritability and depression glossitis referral neuropathy and seborrheic dermatitis Riboflavin vitamin B 12 deficiency is a complication of malabsorption bowel disease and can lead to glossitis and savory dermatologist seborrheic dermatitis B-one deficiency causes beriberi such as peripheral neuropathy with or without cardiac involvement or Warnicke Korsakoff syndrome, and urologic disease most common among patients with alcoholism.b1 deficiency causes beriberi such as peripheral neuropathy with or without cardiac involvement or Warnicke Korsakoff syndrome, and urologic disease most common among patients with alcoholism.

WK3---THURS--->--7yo girl is brought to the ED bc over the past month she has be awakening nightly panicky..sweating..crying and screaming .."Im so scared..monsters are chasing me"..2 months ago she started her new school where she is doing well and making new friends -likely dx?

Nightmare disorder - Nightmare disorder involves recurrent awakenings from rim sleep associated with full alertness and dream recall. It should be differentiated from non-REM sleep terrors, which are characterized by partial arousals unresponsiveness and lack of dream content.

WK3---SUN--->---37yo F at 14wks GA presents for initial prenatal visit...pmh of T2DM and HTN...bp=138/88 and BMI=36...abd non tender and liver is palpated 3cm below the costal margin...labs show APL=145(normal bt 17-88)...AST=115...ALT=125....RUQ US reveals a hyper echoic-appearing liver...several gallstones w/out gb wall thickening and normal appearing cbd... -likely cause of .pts abnormal liver studies?

Nonalcoholic dfatty liver dz -w/ eo DM,htn,obesity..elevated aminotranserases points towards non-alchoilic fatty liver dz...other RFs include T2dm...obesity...dyslipidemia..and metabolic syndrome. Insulin resistance causes unregulated hepatic tag synthesis, peripheral lipolysis and free fatty acid uptake into the liver...all of which promote hepatic fat deposition such as hepatic steatosis...w/ an ast/al;t ratio less than 1. US will show hyper echoic-appearing liver which reflects hepatic fatty infiltration. EO: NAFLD typically presents w/ T2dm..obesity...and elevated aminotransferases w/ ast/alt ratio less than 1. Tx w/ diet...exercise..weightloss...vitamin-E..and pioglitazone

WK3---FRI--->--odds ratio

OR=odd of exposure in cases/odds of exposure in controls -this measure is used case cOntRol studies.

WK3---WED--->--43 yo M presents in the ER barefoot and disheveled...unable to give coherent hx..amd yells"I've been in the Hospital alot..I wont take any meds..you all want to experiment on me." -nbsim?

Offer the pt something to eat and drink Verbal de-escalation techniques should be used to come agitated but non-violent patient and may include the offer of some thing to eat and drink. Intramuscular antipsychotic medication and physical restraints are indicated with verbal de-escalation has failed and violence by a pt is imminent

WK3---SUN--->---25 yo F at 25wks GA presents w/ preterm labor..she and baby's vitals are normal..indomethacin was given for tocolytics... -pt at risk for which of the following OB complications?

Oligohydraminos Indomethacin is good bc it inhibits contractions and delay delivery..BUT...it inhibits COX causing dec in prostaglandin production...and leads to fetal vasoconstriction causing premature closure of ductus arterioles...the vasoconstriction dec renal perfusion and fetal oliguria..resulting in oligo hydramnios(amniotic fluid index 5cm or less)

WK3---SUN--->---43 yo F presents w/ lower abd pain...vomiting...temp=100...sever pain that radiates to LLQ of abd.. -nbsim? -peform diagnostic laparoscopy..

Ovarian torsion or rotation of the ovary around the infundibulopelvic ligament, classically presents w/ nausea..vom...new onset of severe unilateral pain..adnexal tenderness...ovarian torsion is a gynecoplogic emergency and requires diagnostic laparoscopy

WK3---FRI--->--explain per-protocol analysis -what happens to dropout

Per-protocol analysis Epidemiology → Systematic errors Abbreviation: PP analysis A method of statistical analysis in which treatment and control groups are compared using data from only the patients who finished the treatment that they were originally assigned to receive. This method helps to estimate the effect of the treatment itself, but it is prone bias, as the original randomization is no longer preserved. -must exclude drop out from analysis becasue we only want data from ppl who strictly adhered and completed the protocol

WK3---SUN--->---40 yo F presents at 33wks GA...releasing clear fluid..she had polyhydraminos...fetal vitals are fine..tocodynameter reveals contractions every 15-18 mins..US reveals minimal amniotic sac w/ deepest vertical pocket is 1cm -nbsim?

Prophylactic latency antibiotic -the abx is to prevent fulminant intraamniotic infection..inc time bt rupture and delivery EO:uncomplicated preterm prelabor rupture of membranes less than 34wks GA is managed expectantly w/ prophylactic latency antibiotics, steroids and inpatient monitoring. Delivery is at 34wks or sooner if complications develop such as intramniotic infix..placental abruption. 0-amnioinfusion used to tx variable fetal heart rate decelerations in labor.. -tocolysis such as nifedipine—temporarily halts contractions to delay delivery...these are contraindicated in preterm-prelabor-ROM bc contractions often indicate complications such as intramniotic ifnxn or placental abruption that requires delivery or intervention -magnesifum sulfate—given to mom if she's less than 32 wks...for neuroproterction to prevent cerebral palsy -vaginal progesterone...dec risk of preterm delivery in pts w/ incidental shortened cervix(25mm or less)

WK3---SUN--->---35 yo F present for 1st prenatal visit...been trying to conceive for the past 3yrs...pt has been having breast tenderness...abd distention...positive home test...PE reveals a soft non tender tympanic abdomen w/ no masses...ultrasound reveals a thin endometrial stripe(neg pregnancy test) -likely dx?

Pseudocyesis—>a false belief of being pregnant..assc w/ physical signs and sxs of early pregnancy -this likely occurs when the somatization of stress affects the hypothalamic-pituitary axis and causes early pregnancy sxs or when bodily changes such as weight gain/ammenorhea.

WK3---TUE--->-30minut old newborn presents w/ cyanosis...delivered at 32wks..bp=66/42..RR=70..Pulse=160..grunting...intercostal and subcostal retractions..nasal flaring..and shallow respirations -likely dx?

RDS Respiratory distress syndrome classically present in preterm infants with grunting, flaring, retractions, and cyanosis immediately after birth. Chest x-ray shows diffuse ground glass opacities and air bronchograms. Severe coarctation of aorta can present with shock due to poor distal perfusion and closure of the ductus arteriosus. However, the DA begins to constrict as early as 12 hours not immediately after birth. Moreover a lower post doctor such as left lower extremity then preductal such as upper extremity oxygen saturation would be expected. Transposition of the great arteries causes systemic circulation of deoxygenated blood, resulting in cyanosis without retraction or grunting. Supplemental oxygenation does not significantly improve oxygenation. Persistent pulmonary hypertension of the newborn is characterized by high pulmonary vascular resistance, causing right to left shift across the DA. Cyanosis in respiratory distress occur but chest x-ray is often normal and productive oxygen saturation is typically higher than Post Oak to saturation. TTN---causes respiratory distress in the first few hours of life due to delayed resorption of alveolar fluid. Respiratory symptoms and sinuses are typically mild. Chest x-ray reveals perihilar streaking and fluid in the interlobar fissures.

WK3---MON--->-renal vein thrombosis

Renal vein thrombosis Nephrotic syndrome → Complications A complication of hypercoagulable states (e.g., malignancy, nephrotic syndrome, antiphospholipid syndrome) and typically presents with flank pain and hematuria, as well as anuria and elevated creatinine levels if bilateral thrombosis occurs.

WK3---TUE--->--Five year old girl does not talk at school or in social gatherings but interacts with her friends in the neighborhood and siblings at home. What is the likely diagnosis

Selective mutism The patient's refusal to speak in special social situation such as school in contrast to her engagement in normal communication situation in which she feels comfortable such as at home with siblings and friends is consistent with selective mutism. This diagnosis requires one or more months of failure to talk in situations in which it is expected such a school despite the patient oK in other situations. If used to speak at school can impair both academic and social development it should be considered not normal. Children with selective mutism are verbal at home but refused to speak in specific social settings commonly at school it is considered an anxiety disorder and should be treated early to prevent long-term educational hear me. -Children with social pragmatic communication disorder have difficulties with social interaction involving both verbal and number well communication such as eye Contact and body language. Impairment in communication is evident in multiple settings.

WK3---TUE--->--6yo M presents w/ fever..pruritic rash..joint pain..rash began 2days ago and spread to trunks and arms..dx w/ strep pharyngitis 9 days ago..and is on his second to last day of oral penicillin..temp is 101.8 -likely dx?

Serum sickness like reaction Fever itchy rash arthralgia apathy and proteinuria within the first 1 to 2 weeks of exposure to a beta lactam antibiotic or sulfur drug symptoms resolve upon discontinuation of the drug. Serum sickness like reaction is most common in children and is typically triggered by beta lactam such as sickleave for and sulfa drugs. Symptoms began 5 to 14 days after exposure and include itchy rash or brown and low-grade fever. Manifestations resolved with withdrawal of the offending agent Anaphylaxis is IGE mediated type one hypersensitivity reaction that occurs within minutes of drug exposure. Presents with hypertension wheezing gastrointestinal symptoms and or angioedema angioedema

WK3---FRI--->--31 yo F has persistent abd pain for the poast 2 yrs..thats not consistent w/ unrelated meals..multiple exams have been performed and they all come back normal -like dx?

Somatic sxs disorder SSDinvolves excessive anxiety and behaviors related to 1 or more somativ symptoms. These pts typically have high health care and undergo multiple tests w/ negative results. -maligering=external benefit factitious disprder=interal reward of just wanting to assume the sick role

WK3---THURS--->--specific learning disorder(think about academic setting..school age kids)

Specific learning disorder Neurodevelopmental disorders → Specific learning disorder A neurodevelopmental disorder of biological origin that leads to difficulties learning and using academic skills such as reading, spelling, calculating and/or writing. Academic skills are below those expected for the individual's chronological age and interfere with academic, occupational, and daily life. The impairment is not better accounted for by intellectual disabilities, uncorrected visual or auditory acuity, other mental or neurological disorders, lack of proficiency in the language of academic instruction, or inadequate educational instruction, and persists for more than 6 months.

WK3---THURS--->--7yo girl present to office bc refusal to go to school bc she say kids laugh at her...teacher reports the pt get frustrated easily and has difficulty following directions and either makes up word rather than trying sounding them out or angrily refuse to read at all -explanation for pt behavior?

Specific learning disorder...w/ e/o kid struggling to read Learning disorder is present in school age kids as difficulties with academic skills such as reading writing and or mathematics it may result in anxiety behavioral difficulties and school refusal. Early detection and intervention are essential for improving outcomes. Intellectual disability include deficits and intellectual as well as adaptive functioning such as dressing or feeding self, Communicating, socializing.

WK3---WED--->--18yo girl who's been study for final exams present w/ dizziness..palpitations..blood glucose was 48...serum insulin and peptides were drawn together..c-peptide was elevated..has a fam hx of dm -best explains pt's sxs?

Surreptitiously sulfonylurea use Factitious disorder is characterized by intentional falsification of illness in the absence of external reward. Patient appeared to be motivated by internal factors such as psychosocial stressors or desire for validation. Patients with factitious disorder may have hypoglycemia due to SURREPTITIOUS intake ups of sulfonylurea or insulin.

WK3---WED--->--32 yo F w/ pmh of mania presents w/ complaint of facial grimacing and foot tapping..she has been stable for the past 3yrs...taking risperidone an lithium.. -nbsim>?

Taper and discontinue risperidone The patient's facial grimacing combined with tapping and eversion movements are most likely signs of tardive dyskinesia caused by exposure to risperidone a second generation antipsychotic which is used to treat bipolar and psychotic disorders. Patient receiving maintenance antipsychotic medication should be assessed for TD at regular intervals and the causative medication and discontinued it feasible tardive dyskinesia is characterized by abnormal involuntary movements of the face lips tongue trunk or extremities that develop as a result of prolonged exposure to antipsychotic medication. If feasible, the causative medication should be tapered and discontinued. Anticholinergic drugs such as benztropine are indicated to treat acute dystonia. Propranolol has been used in treatment of akathisia, a type of EPS characterized by inner restlessness and inability to sit still.

WK3---SUN--->---32 yo F..24wks GA presents w/ change in bowel habits..for pat 4wks pt had 3 to 4 bloody stools/day..intermittent Lowe abd pain...baby's vitals are fine but is at 12th percentile for size.. -likely dx?

Ulcerative colitis... -hematochezia....abd pain..tenesmus such as fecal urgency followed by straining and inability to defecate most likely has ulcerative colitis..maternal complications can worsen UC..also baby is at risk for preterm delivery and small for gestational age. Most meds used to control UC are safe in pregnancy

WK3---SAT--->---fetal HR w/ VEAL CHOP

VEAL CHOP MINE ..move position..initiate secondary measures...nothing..emergcnecy delivery

WK3---SAT--->---34yo F at 37wks GA presents w/ labor pain...cervix is dilated to 6cm..bf delivery..epidural analgesia was given for pain control...pt began feeling lightheaded and pb dropped to 90/55...pulse 120...RR15.. -likely cause of pt's hypotension

Vasodlation and venous pooling -epidural anesthesia blocks sympathetic nerve fibers responsible for vascular tone in the Lower extremities...resulting in vasodilation(venous pooling)..dec venous return to heart..dec cardiac output...leading to hypotension and compensatory tachycardia. Those w/ significant hypotension can have dec placental infusion such as late decels and fetal hypoxemia EO:hypotensions a common ae of epidural anesthesia, is caused by blood redistribution to the lower extremities bc of venous pooling from sympathetic blockade.

WK3---WED--->--67yo M w/ advanced pancreatic cancer present for f/u stating he no longer wants additional curatives/life prolonging treatment...and wants to avoid hospitalization...he asks if hospice could provide services at home -most appropriate response to pt?

Yes, hospice services are appropriate for you, although you may still need assistance from your family or daily care. Patients with terminal illness wishing to withhold life-prolonging treatments may be appropriate candidate for a hospice care. Home hospice workers can relieve family caregiver burden and provide assistance with medication titration, physical therapy, and palliative medical equipment. The hospice model of care is designed to optimize comfort and quality of life for a patient with a terminal illness this patient is an appropriate candidate for home hospice giving the following criteria terminal illness with six months or less such as advanced pancreatic cancer Patient preference to avoid hospitalization and forgo life-prolonging treatment next pair graph family available at home to provide basic assistance.

WK3---TUE--->-14 y boy presents w/ groin pain..limps...difficulty climbing stairs..+tredelenberg test.,..mild muscular atrophy of the right quadriceps..and gluteal muscles is present... Nbsim?

bilateral hip x-rays Trendelenburg sign is a non-specific finding a gluteal muscle weakness marked by drooping of the contralateral pelvis. SCFE- should be suspected in adolescent who also have chronic hip knee or groin pain limited hip abduction internal rotation and or proximal lower extremity muscle atrophy -Myopathy can lead to proximal muscle weakness and may warrant evaluation with the creatinine kinase level such as beckard muscular dystrophy or skeletal muscular biopsy such as polymyositis.

WK3---FRI--->--a groups of researchers are looking at the assoc bt caffeine consumption and colon cancer of women bt the age of 40-60...p value is 0.02... -whats likely to invaldiate the researcher's conclusion?

generalizability aka the external validity -generalization bc we dont know if the study applies to younger women or older women...bc only middle age women are taking part in the study -internal validity is concerned w/ how well results provide evidence of a causal relationship in a study -reproducibility/reliability focuses on which measure produces consistent results is the measurements are repeated multiple times -statistical power is the ability to detect a siginificant assoc when that associ exists

WK3---THURS--->--69 yo M broke into a store and was caught by police..he told the cops he work there...the store owner denied him..he does provide detail account for past jobs as an English professor..he has a pmh of htn..alcohol abuse Explanation of sxs?

korsakoff syndrome - Korsakoff syndrome is a potential complication of Mauna Kea shuffle apathy that is characterized by retrograde and interior grade amnesia with preserved long-term memory, confabulation, lack of insight, and apathy. Korsakoff syndrome is caused by Diomede efficiency and aerobics mall frequently and alcohol use disorder than an elder malnourished states. Korsakoff is a potential complication of Wernicke encephalopathy acute disorder of thiamine deficiency characterized by the classic triad of a eNCEPHALOPATHY, ataxia and oculomotor dysfunction

WK3---MON--->-3yo boy presents to ED w/ rash on chest..back..arms...had a cough..runny nose...and fever for past week...rash began at head and spread downward...temp is 105..BL eye redness...spots on buccal mucosa... -likely dx?

measles(rubeola).. The program of measles includes fever cough coryza and none purulent conjunctivitis. Koplik spots, small white lesions on the buckle mucosa, may also develop as and or patina Monic of measles. Following the program, a maculopapular rash appears on the face and spread in cephalocaudal and centrifugal pattern..tx w/vitamin A Adeno virus which can cause pharyngoconjunctival fever presents with pharyngitis, nonpurulent conjunctivitis, and fever. Erythema infectiosum/5th disease is caused by human Parville be nice teen. Children develop fever and I slapped cheek rash. Kawasaki disease is characterized by fever for more than five days mucous membrane changes extremities changes none exudative conjunctivitis, cervical lymph nodes greater than 1.5 cm, and polymorphous rash. Roseola is caused by human herpesvirus six. He presents with fever peaks with rash on said the maculopapular rash roseola appears as the fever resolves Rubella presents similar to measles but is relatively mild such as low-grade fever and patient is well appearing and it has a shorter duration. Rubella rash spread cephalocaudal but does not darken as does measles. Scarlett fever is caused by strep pyogenes and present with pharyngitis fever and erythematous sandpaper like rash.

WK3---FRI--->--measurement bias

occurs from poor data collection w/ inaccurate results

WK3---FRI--->--selection bias

occurs when samples or participants are selected that differ from other groups in additional determinants of outcome

WK3---FRI--->--80 yo F w/ multiple medical problems is admitted to the hospital for evaluation of syncope..pt is taking diltiazem...metoprolol...and antenolol...which medication reconciliaiton interventions is most likely to result in a decreased in adverse drug events and health care utilization?

pharmacist directed interventions -continuity of meds at the time of transition of care bt inpatient and outpatient facilities and w/ inpatient facilities is potential source of medical error. Interventions that target pharmacy pesonell and high risk patients appear to be most effective in improving the quality of patient care.

WK3---FRI--->--recall bias

pt w/ dz are more likely to recall exposures..

WK3---FRI--->--define false neg

pt who have neg test result despite actually having the dz -moving from A to B will allow the FN to increase

WK3---THURS--->--23yo M brought to the office by his parentsdt concerns that he become increasingly depressed and withdrawn over the past yr...pt dropout of school...since his breakup w/ his girlfriend...and stopped going out w/ friends...been watching the same video for the past few months to check for signals -like ly dx?

schizophrenia The patient's delusions receiving special messages from videos being secretly recorded negative symptoms Cygnet significant functional decline and continuous impairment for six more months are consistent with schizophrenia other common hallucinations are not required for the diagnosis this patient's negative symptoms include apathy avolition affective flattening facial expressions very brief answers socially withdrawn. Delusional disorder is characterized by one or more delusion in the absence of other psychotic symptoms. Functioning is normal apart from the direct impact of the delusion. A diagnosis of delusion disorder A major depressive disorder with psychotic features, which is a severe subtype of major depressive disorder, the patient develops delusions and or hallucinations, typically with depressive themes of guilt and worthlessness. This obsessive compulsive disorder obsession such as thoughts images and urges and compulsion such as behavior and mental ask and that relieving distress are necessary to diagnose obsessive compulsion disorder Patients paranoid personality disorder are generally suspicious and distrustful and patience with schizoid personality disorder or emotionally detached

WK3---FRI--->--which parameters of a study will inc if you decrease the sample size?

studies w/ larger sample sizes have a greater power..such as a false H0 is rejected..and therefore lower probability of a type 2 error than do studies w/ smaller sample sizes

WK3---FRI--->--ppv and NPV

the probability that a person w/ a positive test truly has the dz..the higher the dz prevalence..the higher the PPV PPV=TP/TP+FP NPV=the probab that a person w a neg test truly does not have the dz...the lower the dz prevalence..the higher the NPV NPV=TN/TN+FN predictive values depend on the prevalence of dz in a sample


Kaugnay na mga set ng pag-aaral

Ricci → Ch. 1: Perspectives on Maternal and Child Health Care PrepU

View Set

Enfermedad inflamatoria intestinal

View Set

TExES Core Subjects EC-6: Economic Principles

View Set

ACCT 202 - UNL - SmartBook Unit 4

View Set